Vous êtes sur la page 1sur 64

TIRAZONA V. PHIL EDS TECHNO-SERVICE INC G.R.

NO 169712 (2009)
FACTS: Tirazona worked as administrative manager of PET. After PET
officers/directors called her attention to her improper handling of a situation involving a
rank-and-file employee, she claimed that she was denied due process for which she
demanded indemnity from PET.
1. SC denied her claims for illegal termination on the account her arrogance,
hostility and uncompromising attitude justify the companys decision to terminate
her employement
2. Tirazona filed a MR citing that SC failed to consider the length of her service to
PET in affirming her termination from employment. She prayed that her dismissal
be declared illegal. Should Sc uphold the legality of her dismissal, Tirazona
pleaded she be awarded separation pay and retirement benefits out of
humanitarian considerations
ISSUE: WON THE PETITIONER IS ENTITLED TO SEPARATION PAY AND
RETIREMENT BENEFITS
HELD: No, as a general rule an employee who has been dismissed for any of the just
causes enumerated under Art 282 Labor Code is not entitled to separation pay.
Separation pay shall only allowed as a measure of social justice only in those instances
where the employee is validly dismissed for causes other than serious misconduct or
those reflecting on his moral character.
The policy of social justice is not intended to countenance wrongdoing simply because it
is committed by the underprivileged. At best it may mitigate the penalty but it certainly
will not condone the offense. Compassion for the poor is an imperative of every humane
society but only when the recipient is not a rascal claiming an undeserved privilege.
Social justice cannot be permitted to be [a] refuge of scoundrels any more than can
equity be an impediment to the punishment of the guilty. Those who invoke social
justice may do so only if their hands are clean and their motives blameless and not
simply because they happen to be poor. This great policy of our Constitution is not
meant for the protection of those who have proved they are not worthy of it, like the
workers who have tainted the cause of labor with the blemishes of their own character.

SALINAS V NLRC 319 SCRA 54 (1999)


FACTS: Petitioners were employed with Atlantic Gulf and Pacific Co. (AG & P): Salinas:
1983-1988 as carpenter/finishing carpenter; Alejandro: 1982-1989 as bulk cement
operator, bulk cement plant/carrier operator & crane driver; Cortez: 1979-1988 as
carpenter/forklift operator and; Samulde: 1982-1989 as lubeman/stationary operator
1. Complaints (separate but consolidated by the LA): illegal dismissal
2. Petitioners Claim: They had been covered by a number of contracts renewed
continuously, with periods ranging from five (5) to nine (9) years, and they
performed the same kind of work through out their employment, and such was
usually necessary and desirable in the trade or business of the respondent
corporation; and their work did not end on a project-to-project basis, although the
contrary was made to appear by the employer through the signing of separate
employment contracts.
3. LA: Dismissed petitions on the ground that the petitioners are project employees
are project employees whose work contracts with AG & P indicate that they were
employed in such category; that they have been assigned to different work
projects, not just to one and that their work relation with AG & P, relative to
termination, is governed by Policy Instruction No. 20 (rule governing project
employees).
4. Appeal to NLRC: Affirmed LAs findings
ISSUES: WON THE PETITIONERS ARE PROJECT EMPLOYEES
HELD: No, the petitioners are regular employees. he mandate in Art 281 Labor Code,
which pertinently prescribes that the 'provisions of written agreement to the contrary
notwithstanding and regardless of the oral agreements of the parties, an employment
shall be deemed to be regular where the employee has been engaged to perform
activities which are usually necessary or desirable in the usual business or trade of the
employer' and that any employee who has rendered at least one year of service,
whether such service is continuous or broken shall be considered a regular employee
with respect to the activity in which he is employed and his employment shall continue
while such actually exists,' should apply in the case of petitioner.
Failure to report the termination to Public Employment Office is a clear indication that
petitioners were not and are not project employees. (PI No. 20 requires reports of
terminations)
It is basic and irrefragable rule that in carrying out and interpreting the provisions of the
Labor Code and its implementing regulations, the workingman's welfare should be the
primordial and paramount consideration. The interpretation herein made gives meaning
and substance to the liberal and compassionate spirit of the law enunciated in Article 4
of Labor Code that "all doubts in the implementation and interpretation of the provisions
of the Labor Code including its implementing rules and regulations shall be resolved in
favor of labor".

It is beyond cavil that petitioners had been providing the respondent corporation with
continuous and uninterrupted services, except for a day or so gap in their successive
employment contracts. Their contracts had been renewed several times, with the total
length of their services ranging from five (5) to nine (9) years. Throughout the duration
of their contracts, they had been performing the same kinds of work (e.g., as lubeman,
bulk cement operator and carpenter), which were usually necessary and desirable in the
construction business of AG & P, its usual trade or business.
Undoubtedly, periods in the present case have been imposed to preclude the
acquisition of tenurial security by petitioners, and must be struck down for being
contrary to public policy, morals, good customs or public order.

LEDESMA JR V. NLRC 537 SCRA 358 (2007)


FACTS: Petitioner Ledesma was employed as a bus/service driver by the private
respondent Phil National Training Institute (PNTI) on probationary basis. As such he
was required to report at private respondents training site in Dasmarinas, Cavite under
the direct supervision of its site administrator, Pablo Manolo de Leon
1. Petitioner filed a complaint against de Leon for allegedly abusing his authority as
site administrator by using PNTIs vehicles and other facilities for personal ends.
Ledesma also accused de Leon of immoral conduct allegedly carried out within
PNTIs premises.
2. De Leon filed a written report against petitioner citing his suspected drug use. As
such the HR manager served a notice to explain to petitioner for violating the
companys code of conduct.
3. Subsequently, petitioner filed a complaint for illegal dismissal before LA, alleging
that the report regarding his suspected drug use was means of retaliation against
him. Petitioner also averred that he was barred from entering the work place
when he reported to work after undergoing a drug test which he personally
requested
4. Private respondent, on the other hand, countered that petitioner was never
dismissed from employed but was merely served a notice to explain in view of his
suspected drug use. But instead of filing an answer to the said notice, petitioner
prematurely lodged a complaint for illegal dismissal against PNTI
5. LA held in favor of petitioner, declaring his dismissal as illegal. However NLRC
reversed LA decision citing that petitioner failed to establish the fact of dismissal
for his claims
ISSUE: WON PETITONER WAS ILLEGALLY DISMISSED
HELD: No, in the present case there is hardly any evidence on record so as to meet the
quantum of evidence required. Petitioners claim of illegal dismissal is supported by no
other than his own bare, uncorroborated and self-serving allegations, which are
incoherent and inconsistent.
While SC is not unmindful of the rule that in cases of illegal dismissal, the employer
bears the burden of proof to prove that the termination was for a valid or authorized
cause in the case at bar, however, the facts and the evidence did not establish a prima
facie case that the petitioner was dismissed from employment. Before the private
respondent must bear the burden of proving that the dismissal was legal, petitioner
must first establish by substantial evidence the fact of his dismissal from service.
Logically, if there is no dismissal, then there can be no question as to the legality or
illegality thereof.
In earlier cases, the SC held that the burden of proving allegations rest upon the party
alleging, to wit:
1. The rule is that one who alleges a fact has the burden of proving it; thus,
petitioners were burdened to prove their allegation that respondents dismissed

them from their employment. It must be stressed that the evidence to prove
this fact must be clear, positive and convincing. The rule that the employer
bears the burden of proof in illegal dismissal cases finds no application here
because the respondents deny having dismissed the petitioners
2. It is a basic rule in evidence, however, that the burden of proof is on the part of
the party who makes the allegations ei incumbit probatio, qui dicit, non qui
negat. If he claims a right granted by law, he must prove his claim by
competent evidence, relying on the strength of his own evidence and not
upon the weakness of that of his opponent
It is true that the Constitution affords full protection to labor, and that in light of
Constitutional mandate, we must be vigilant in striking down any attempt of
management to exploit or oppress the working class. However, it does not mean
we are bound to uphold the working class in every labor dispute brought before
Court for our resolution.

this
the
that
this

The law in protecting the rights of the employees, authorizes neither oppression nor
self-destruction of the employer. It should be made clear that when the law tilts the
scales of justice in favor of labor, it is in recognition of the inherent economic inequality
between labor and management. The intent is to balance the scales of justice; to put the
two parties on relatively equal positions. There may be cases where the circumstances
warrant favoring labor over the interests of management but never should the scale be
so tilted if the result is an injustice to the employer. Justitia nemini neganda est -- justice
is to be denied to none.

ANGELES CONSTRUCTION V. NLRC 305 SCRA 734 (1999)


FACTS: Private respondent Pedro Santos was employed in 1969 as a carpenter, by the
petitioner JV Angeles Construction Corp. In 1973, he was promoted to a position of
foreman which he held until his retirement in February 1992 when he was 62 years old
1. Santos filed a complaint for retirement benefits and service incentive leave pay.
LA held in favor of petitioner
2. Petitioner filed an appeal before NLRC, assailing that the said ruling of LA
granting retirement benefits to Santos, by giving RA 7641 (Retirement Pay Law)
a retroactive application although Santos had retired almost a year prior to the
effectivity of said law in January 7, 1993. Petitioner cited the case of Llora Motors
v. Drilon where the SC held that in the absence of a CBA or other employment
contract, there is no obligation on the part of the employer to set up a retirement
scheme over and above that already established under existing laws. Since
Santos has been receiving his retirement benefits from SSS, he cannot anymore
ask for additional benefits from his employer in the absence of company practice,
policy or contract granting such benefits
3. NLRC upheld LA decision citing Oro Enterprises v. NLRC
ISSUE: WON RA 7641 CAN BE RETROACTIVELY APPLIED IN THIS CASE
HELD: No. In Oro Enterprises, Inc. v. NLRC, the court held that R.A. 7641 can be
applied retroactively, rationalizing thus: "R.A. 7641 is undoubtedly, a social legislation.
The law has been enacted as a labor protection measure and as a curative statue that absent a retirement plan devised by, an agreement with, or a voluntary grant from an
employer can respond, in part at least to the financial well-being of workers during
their twilight years soon following their life of labor. There should be little doubt about
the fact that the law can apply to labor contracts still existing at the time the statute has
taken effect, and that its benefits can be reckoned not only from the date of the laws
enactment but retroactively to the time said employment contracts have started.
CJC Trading Inc. v. NLRC, enumerated the circumstances which must occur before the
law could be given retroactive effect, to wit: (1) the claimant for retirement benefits was
still the employee of the employer at the time the statute took effect; and (2) the
claimant has complied with the requirements for eligibility under the statute for such
retirement benefits.
In the case under scrutiny, private respondent Santos retired and ceased to be an
employee of petitioner on February 1992, eleven (11) months before the effectivity
of R.A. 7641, and he brought his complaint on October 23, 1993, nine (9) months
after the laws effectivity. It is thus decisively clear that the provisions of R.A.
7641 could not be given retroactive effect in his favor. Consequently, the NLRC
erred in upholding the Labor Arbiters award of retirement benefits to private
respondent.

ROSARIO BROS V. OPLE 131 SCRA 72 (1984)


FACTS: Private respondents are tailors, pressers, stitchers and similar workers hired by
the petitioner in its tailoring department (Modes Suburbia). Some had worked there
since 1969 until their separation in January 1978.
1. For their services, they were paid weekly wages on piece-work basis, minus the
withholding tax. Further, they were registered with SSS as employees of
petitioner and premiums were deducted from their, wages; they were also
members of the Avenida-Cubao Manila COD Department Store Labor Union
which has a CBA with the company and; they were required to report for work
from Monday to Saturday and to stay in the tailoring shop for no less than 8
hours a day, unless no job order was given them after waiting for 2-3 hours. Their
attendance was recorded through a bundy clock just like other employees of
petitioner. A master cutter distributes job orders equally, supervises the work and
sees to it that they were finished as a soon as possible
2. Subsequently, private respondents filed a complaint for violation of PD 851 (13 th
month pay) and PD 525 (Emergency living allowance) against petitioner
3. LA rendered in favor of petitioner, citing that private respondents are not
employees of the petitioner within the meaning of Art 267(b) Labor Code. NLRC
reversed the same
ISSUE: WON THERE IS AN EMPLOYER-EMPLOYEE RELATIONSHIP BETWEEN
PETITIONER AND PRIVATE RESPONDENTS
HELD: Yes, As held in Mafinco Trading Corporation vs. Ople, 70 SCRA 139, the
existence of employer-employee relationship is determined by the following elements,
namely: (1) the selection and engagement of the employee; (2) the payment of wages;
(3) the power of dismissal; and (4) the power to control employees' conduct although
the latter is the most important element. On the other hand, an independent contractor
is one who exercises independent employment and contracts to do a piece of work
according to his own methods and without being subjected to control of his employer
except as to the result of his work.
1. In the case at bar, as found by the public respondent, the selection and hiring of
private respondents were done by the petitioner, through the master cutter of its
tailoring department who was a regular employee. The procedure was modified
when the employment of personnel in the tailoring department was made by the
management itself after the applicants' qualifications had been passed upon by a
committee of four. Later, further approval by the Personnel Department was
required.
2. Private respondents received their weekly wages from petitioner on piece-work
basis which is within the scope and meaning of the term "wage" as defined under
Article 97(f) Labor Code
3. Petitioner had the power to dismiss private respondents, as shown by the various
memoranda issued for strict compliance by private respondents, violations of
which, in extreme cases, are grounds for outright dismissal. In fact, they were
dismissed on January 2, 1978, although, the dismissal was declared illegal by

LA.
4. Private respondents' conduct in the performance of their work was controlled by
petitioner, such as: (1) they were required to work from Monday through
Saturday; (2) they worked on job orders without waiting for the deadline; (3) they
were to observe cleanliness in their place of work and were not allowed to bring
out tailoring shop patterns; and (4) they were subject to quality control by
petitioner.
5. Private respondents were allowed to register with the Social Security System
(SSS) as employees of petitioner and premiums were deducted from their wages
just like its other employees. And, withholding taxes were also deducted from
their wages for transmittal to the Bureau of Internal Revenue (BIR).
Well-established is the principle that "findings of administrative agencies which have
acquired expertise because their jurisdiction is confined to specific matters are generally
accorded not only respect but even finality. Judicial review by this Court on labor cases
do not go so far as to evaluate the sufficiency of the evidence upon which the Deputy
Minister and the Regional Director based their determinations but are limited to issues
of jurisdiction or grave abuse of discretion." In the case at bar, the questioned decision
and order of execution of public respondents are not tainted with unfairness or
arbitrariness that would amount to abuse of discretion or lack of jurisdiction and,
therefore, this Court finds no necessity to disturb, much less, reverse the same.

ZANOTTE SHOES V. NLRC 241 SCRA 261 (1961)


FACTS: Private respondents filed a complaint for illegal dismissal and various money
claims against petitioners.
1. They alleged that they worked for a minimum of 12 hours daily, including
Sundays and holidays when needed; they were paid on piece-work basis
2. They also alleged that it angered petitioner when they requested to be made
members of SSS and when they demanded an increase in their pay rates, they
were prevented from entering the work premises
3. Petitioners, on the other hand, claimed that their business operations were only
seasonal, normally twice a year (one in June and another in December), when
heavy job orders would come in. According to petitioners, private respondents
were engaged on purely contractual basis
4. LA rendered in favor of complainants, citing that there was an employeremployee relationship that existed between the two. NLRC affirmed the same
ISSUE: WON THERE EXISTS AN EMPLOYER-EMPLOYEE
BETWEEN PETITIONER AND PRIVATE RESPONDENTS

RELATIONSHIP

HELD: Yes. The work of private respondents is clearly related to, and in the pursuit of,
the principal business activity of petitioners. The indicia used for determining the
existence of an employer-employee relationship, all extant in the case at bench, include
(a) the selection and engagement of the employee; (b) the payment of wages; (c) the
power of dismissal; and (d) the employer's power to control the employee with respect
to the result of the work to be done and to the means and methods by which the work to
be done and to the means and methods by which the work is to be accomplished. The
requirement, so herein posed as an issue, refers to the existence of the right to control
and not necessarily to the actual exercise of the right.
While this Court up holds the control test under which an employer-employee
relationship exists "where the person for whom the services are performed reserves a
right to control not only the end to be achieved but also the means to be used in
reaching such end," it finds no merit with petitioner's arguments as stated above. It
should be borne in mind that the control test calls merely for the existence of the right to
control the manner of doing the work, not the actual exercise of the right.

DEALCO FARMS V. NLRC G.R. NO 153192 (2009)


FACTS: Petitioner is a corporation engaged in the business of importation, production,
fattening and distribution of live cattle for sale to meat dealers, meat traders, meat
processors, canned goods manufacturers and other dealers in Mindanao and Metro
Manila
1. Private respondents were hired by petitioner as escorts or comboys for the
transit of live cattle from General Santos City to Manila. Respondents work
entailed tending to the cattle during transportation. It included feeding and
frequently showering the cattle to prevent dehydration and to develop heat
resistance.
2. Upon arrival to Manila, the cattle are turned over to and received by duly
acknowledged buyers or customers of petitioner, at which point, respondents
work ceases. For every round trip travel which lasted an average of 12 days,
respondents were each paid P1500. The 12-day period is occasionally extended
when petitioners customers were delayed in receiving the cattle.
3. Private respondents filed a complaint for illegal dismissal with claims for
separation pay with full back wages, salary differentials, service incentive leave
pay, 13th month pay etc.
4. Respondents alleged that: 1. They were illegally dismissed, as they never
violated any of petitioners company rules and policies; 2. Their dismissal was not
due to any just or authorized cause and; 3. petitioner did not observe due
process in effecting their dismissal, failing to give them written notice thereof
5. Petitioner, however, asserted that the finished cattle are sold to traders and
middlemen who undertake transportation thereof to Manila. Buyers themselves
arrange, through local representatives, for the hauling from petitioners farm to
the port area, shipment of the finished cattle to Manila and; escort or comboy
services to feed and water the cattle during transit. There was only once instance
when it engaged the services of respondents as comboys which was only on a
per-trip or per-contract basis
6. Petitioner posited that respondents are independent contractors who offer
comboy services to various shippers and traders of cattle, not only to petitioner;
in the performance of work on board the ship, respondents are free from the
control and supervision of the cattle since the latter is interested only in the result
thereof; in the alternative, respondents can only be considered as casual
employees performing work not necessary and desirable to the usual business of
petitioner; respondents likewise failed to complete the one-year service period,
whether continuous or broken, set forth in Art 280 Labor Code as petitioners
shipments were substantially reduced in 1988-89, thereby limiting the escort
activity for which respondents were employed
7. LA found that private respondents were employees of Dealco Farms since all the
four elements in the determination of an employer-employee relationship were
present in the instant case. NLRC affirmed the same
ISSUE: WON PRIVATE RESPONDENTS ARE EMPLOYEES OF DEALCO FARMS

HELD: Yes, the presence of the 4 elements in the determination of employer-employee


relationship has been clearly established by the facts and evidence on record, starting
with the admissions of petitioner who acknowledged the engagement of respondents as
escorts of their cattle from General Santos City to Manila with compensation of P1500
per trip.
In determining the existence of an employer-employee relationship between the parties,
both the Labor Arbiter and the NLRC examined and weighed the circumstances against
the four-fold test which has the following elements: (1) the power to hire, (2) the
payment of wages, (3) the power to dismiss, and (4) the power to control the
employees conduct, or the so-called "control test." Of the four, the power of control is
the most important element. More importantly, the control test merely calls for the
existence of the right to control, and not necessarily the exercise thereof.
Having failed to substantiate its allegation on the relationship between the parties, we
stick to the settled rule in controversies between a laborer and his master that doubts
reasonably arising from the evidence should be resolved in the formers favor. The
policy is reflected in no less than the Constitution, Labor Code and Civil Code.

METEORO ET AL V. CREATIVE CREATURES G.R. NO 171275 (2009)


FACTS: Respondent Creative Creatures Inc is a corporation engaged in the business of
producing, providing, or procuring the production of set designs and set construction
services for TV exhibitions, concerts, theatrical performances, motion pictures and the
like. It primarily caters to the production design requirements of ABS-CBN in Metro
Manila and nationwide.
1. On the other hand, petitioners were hired by respondent on various dates as
artists, carpenters and welders. They were tasked to design, create, assemble,
set up and dismantle props, and provide sound effects to respondents various
TV programs and movies
2. Petitioners filed a complaint for nonpayment of night shift differential pay,
overtime pay, holiday pay, 13th month pay, premium pay for Sundays and/or rest
days, service incentive leave pay, paternity leave pay, educational assistance,
rice benefits, and illegal/unauthorized deductions from salaries
3. Respondent claimed that petitioners were contractual employees and/or
independent talent workers and that DOLE had no jurisdiction over the complaint
because of the absence of an employer-employee relationship
4. DOLE held in favor petitioners, citing that the four elements in determination an
employer-emmployee relationship were present in the instant case
5. CA reversed the decision and held in favor of respondent Creative Creatures
based on the exception provided in Art 128 Labor Code
ISSUE: WON DOLE HAS JURISDICTION OVER THE INSTANT CASE
HELD: No, The last sentence of Article 128 (b) of the Labor Code, otherwise known as
the exception clause, provides an instance when the Regional Director or his
representatives may be divested of jurisdiction over a labor standards case.
Under prevailing jurisprudence, the so-called exception clause has the following
elements, all of which must concur:
(a) that the employer contests the findings of the labor regulations officer
and raises issues thereon;
(b) that in order to resolve such issues, there is a need to examine
evidentiary matters; and
(c) that such matters are not verifiable in the normal course of inspection.
In the present case, the CA aptly applied the exception clause. At the earliest
opportunity, respondent registered its objection to the findings of the labor inspector.
The labor inspector, in fact, noted in its report that respondent alleged that petitioners
were contractual workers and/or independent and talent workers without control or
supervision and also supplied with tools and apparatus pertaining to their job. In its
position paper, respondent again insisted that petitioners were not its employees. It
then questioned the Regional Directors jurisdiction to entertain the matter before it,
primarily because of the absence of an employer-employee relationship. Finally, it

raised the same arguments before the Secretary of Labor and the appellate court. It is,
therefore, clear that respondent contested and continues to contest the findings and
conclusions of the labor inspector.
Thus, in addition to the above-mentioned documents, other pieces of evidence are
considered in ascertaining the true nature of the parties relationship. This is especially
true in determining the element of control. The most important index of an employeremployee relationship is the so-called control test, that is, whether the employer
controls or has reserved the right to control the employee, not only as to the result of the
work to be done, but also as to the means and methods by which the same is to be
accomplished.
In the case at bar, whether or not petitioners were independent contractors/project
employees/free lance workers is a question of fact that necessitates the examination of
evidentiary matters not verifiable in the normal course of inspection. Indeed, the
contracts of independent services, as well as the check vouchers, were kept and
maintained in or about the premises of the workplace and were, therefore, verifiable in
the course of inspection. However, respondent likewise claimed that petitioners were
not precluded from working outside the service contracts they had entered into with it
(respondent); and that there were instances when petitioners abandoned their service
contracts with the respondent, because they had to work on another project with a
different company.

DOMONDON V. NLRC 471 SCRA 559 (2005)


FACTS: Domondon filed a complaint against private respondent Van Melle Phils Inc
(VMPI), claiming illegal dismissal and thus prayed for reinstatement, payment of full
back wages inclusive of allowances, 14th month pay, sick and vacation leaves, share in
the profits etc.
1. Petitioner alleged that VMPI hired him as a materials manager through the
companys president and GM Victor Endaya. He was given a guaranteed
monthly salary of P98,000 for 14 months with annual merit adjustment, profit
sharing bonus from 0-2 months based on individual, company and corporate
performance and a brand new car with gas allowance
2. Petitioner averred that Have as the new president of VMPI requested his
courtesy resignation and when he refused, life got difficult for him. His decisions
were always questioned Have and was also subjected to verbal abuse
3. On their part, private respondents admitted hiring petitioner but denied illegally
dismissing him. According to the private respondents, Domondon informed them
about his intention to resign and requested a soft land financial support for
P300,000. Petitioner also proposed the transfer of ownership of the car assigned
to him in lieu of the financial assistance from the company. Since company policy
prohibits disposition of assets without valuable consideration, the parties agreed
that petitioner shall pay for the car with the soft landing financial assistance from
VMPI
4. VMPI alleged that when petitioner received the P300,000, he did not use it to pay
for the car as agreed upon. Repeated demands for payment were unheeded.
VMPI, then, gave petitioner an option to apply his sick and vacation leave credits,
13th and 14th month pay less taxes and pay the balance thereof, or return the car
to the company. Petitioner did not exercise either option and instead filed a
complaint for illegal dismissal
5. LA held in favor of VMPI and dismissed the case for lack of merit. NLRC affirmed
the same
ISSUE: WON THE TRANSFER OF OWNERSHIP OF TH SAID CAR IS WITHIN THE
JURISDICTION OF NLRC
HELD: The jurisdiction of Labor Arbiters is provided under Article 217(a) of the Labor
Code, as amended:
(a) Except as otherwise provided under this Code the Labor Arbiters shall
have original and exclusive jurisdiction to hear and decide, within thirty
(30) calendar days after the submission of the case by the parties for
decision without extension, even in the absence of stenographic notes, the
following cases involving all workers, whether agricultural or nonagricultural:
1.
2.

Unfair labor practice cases;


Termination disputes;

3.

4.
5.

6.

If accompanied with a claim for reinstatement, those cases that


workers may file involving wages, rates of pay, hours of work
and other terms and conditions of employment;
Claims for actual, moral, exemplary and other forms of
damages arising from employer-employee relations;
Cases arising from any violation of Article 264 of this Code,
including questions involving the legality of strikes and
lockouts;
Except claims for Employees Compensation, Social Security,
Medicare and maternity benefits, all other claims, arising from
employer-employee relations, including those of persons in
domestic or household service, involving an amount exceeding
five thousand pesos (P5,000.00) regardless of whether
accompanied with a claim for reinstatement.

In all these instances, the matrix is the existence of an employer-employee relationship.


In the case at bar, there is no dispute that petitioner is an employee of the respondents.
In the case at bar, petitioner claims illegal dismissal and prays for reinstatement,
payment of full back wages inclusive of allowances, 14 th month pay, sick and vacation
leaves, share in the profits, moral and exemplary damages and attorneys fees. These
causes of action clearly fall within the jurisdiction of LA, specifically under paragraphs 2,
3 and 4 of Article 217(a). On the other hand, private respondents made a counterclaim
involving the transfer of ownership of a company car to petitioner. They maintain that
he failed to pay for the car in accordance with their agreement.
Based on the facts presented, the transfer of the ownership of the company car to
petitioner is connected with his resignation and arose out of the parties employeremployee relations. Accordingly, private respondents claim for damages falls within the
jurisdiction of the Labor Arbiter.

EVIOTA V. CA 407 SCRA 394 (2003)


FACTS: Respondent Standard Chartered Bank and petitioner Eviota executed a
contract of employment where the latter was employed as compensation and benefits
manager. However, petitioner abruptly resigned from the respondent bank barely a
month after his employment and rejoined his former employer
1. The bank alleged that Eviota made off with a files and other documents
containing confidential information on employee compensation and other bank
matters
2. Petitioner filed a motion to dismiss on the ground that the action for damages
was within the exclusive jurisdiction of LA under par 4 Art 217 Labor Code.
Petitioner alleged that the banks claim for damages arose out of or were in
connection with his employer-employee relationship with the respondent bank
3. Trial court denied petitioners motion to dismiss citing that the relief prayed for by
Standard Chartered Bank was ground on the tortious manner by which petitioner
terminated his employment with the latter, and as such is governed by NCC. CA
affirmed the same
ISSUE: WON THE LA HAS JURISDICTION OVER THE CASE
HELD: Not every controversy or money claim by an employee against the employer or
vice-versa is within the exclusive jurisdiction of the labor arbiter. A money claim by a
worker against the employer or vice-versa is within the exclusive jurisdiction of the labor
arbiter only if there is a reasonable causal connection between the claim asserted and
employee-employer relation. Absent such a link, the complaint will be cognizable by the
regular courts of justice.
Actions between employees and employer where the employer-employee relationship is
merely incidental and the cause of action precedes from a different source of obligation
is within the exclusive jurisdiction of the regular court. In Georg Grotjahn GMBH & Co.
v. Isnani, we held that the jurisdiction of the Labor Arbiter under Article 217 of the Labor
Code, as amended, is limited to disputes arising from an employer-employee
relationship which can only be resolved by reference to the Labor Code of the
Philippines, other labor laws or their collective bargaining agreements.
The claims were the natural consequences flowing from a breach of an obligation,
intrinsically civil in nature.
In this case, the private respondents first cause of action for damages is anchored on
the petitioners employment of deceit and of making the private respondent believe that
he would fulfill his obligation under the employment contract with assiduousness and
earnestness. The petitioner volte face when, without the requisite thirty-day notice
under the contract and the Labor Code of the Philippines, as amended, he abandoned
his office and rejoined his former employer; thus, forcing the private respondent to hire a
replacement. The private respondent was left in a lurch, and its corporate plans and
program in jeopardy and disarray. Moreover, the petitioner took off with the private

respondents computer diskette, papers and documents containing confidential


information on employee compensation and other bank matters. On its second cause
of action, the petitioner simply walked away from his employment with the private
respondent sans any written notice, to the prejudice of the private respondent, its
banking operations and the conduct of its business. Anent its third cause of action, the
petitioner made false and derogatory statements that the private respondent reneged on
its obligations under their contract of employment; thus, depicting the private
respondent as unworthy of trust.
It is evident that the causes of action of the private respondent against the
petitioner do not involve the provisions of the Labor Code of the Philippines and
other labor laws but the New Civil Code. Thus, the said causes of action are
intrinsically civil. There is no causal relationship between the causes of action of
the private respondents causes of action against the petitioner and their
employer-employee relationship. The fact that the private respondent was the
erstwhile employer of the petitioner under an existing employment contract
before the latter abandoned his employment is merely incidental.

ALVIADO ET AL V. PROCTER & GAMBLE AND PROMM-GEM INC G.R. NO 160506


(2010)
FACTS: Petitioners worked as merchandisers of P&G from various dates, allegedly
starting as early as 1982 or as late as June 1991 to either May 1992 or March 1993.
1. Petitioners filed a complaint against P&G for regularization, service incentive
leave pay and other benefits; the complaint was later amended to include illegal
dismissal
2. LA dismissed the complaint for lack of merit and ruled that there was no
employer-employee relationship between petitioners and P&G as the selection
and engagement of petitioners, payment of wages, power of dismissal and
control with respect to the means and methods by which their work was
accomplished, were all done and exercised by Promm-Gem Inc. Promm-Gem Inc
was a legitimate independent job contractor. NLRC affirmed the same
3. Petitioners claimed they were recruited by the salesmen of P&G and were
engaged to undertaking merchandising chores for P&G even before the
existence of Promm-Gem. Petitioners also alleged that P&G instigated their
dismissal from work. Petitioners further assert that Promm-Gem are labor-only
contractors providing services of manpower to their client
4. P&G, on the other hand, argued that there is no employment relationship
between them and petitioners. P&G also contends that Labor Code neither
defines nor limits which services or activities may be validly outsourced. Thus, an
employer can farm out any of its activities to an independent contractor,
regardless of whether such activity is peripheral or core in nature. It insisted that
the determination of whether to engage the services of a job contractor or to
engage in direct hiring is within the ambit of management prerogative
ISSUE: WON P&G IS THE EMPLOYER OF PETITIONERS
HELD: In order to resolve the issue of whether P&G is the employer of petitioners, it is
necessary to first determine whether Promm-Gem and SAPS are labor-only contractors
or legitimate job contractors.
There is "labor-only" contracting where the person supplying workers to an employer
does not have substantial capital or investment in the form of tools, equipment,
machineries, work premises, among others, and the workers recruited and placed by
such person are performing activities which are directly related to the principal business
of such employer. In such cases, the person or intermediary shall be considered merely
as an agent of the employer who shall be responsible to the workers in the same
manner and extent as if the latter were directly employed by him.
Clearly, the law and its implementing rules allow contracting arrangements for the
performance of specific jobs, works or services. Indeed, it is management prerogative to
farm out any of its activities, regardless of whether such activity is peripheral or core in
nature. However, in order for such outsourcing to be valid, it must be made to an

independent contractor because the current labor rules expressly prohibit labor-only
contracting.
To emphasize, there is labor-only contracting when the contractor or sub-contractor
merely recruits, supplies or places workers to perform a job, work or service for a
principal and any of the following elements are present:
i) The contractor or subcontractor does not have substantial capital or
investment which relates to the job, work or service to be performed and the
employees recruited, supplied or placed by such contractor or subcontractor
are performing activities which are directly related to the main business of the
principal; or
ii) The contractor does not exercise the right to control over the performance of
the work of the contractual employee. Under the circumstances, Promm-Gem
cannot be considered as a labor-only contractor. We find that it is a legitimate
independent contractor.
Where labor-only contracting exists, the Labor Code itself establishes an employeremployee relationship between the employer and the employees of the labor-only
contractor." The statute establishes this relationship for a comprehensive purpose: to
prevent a circumvention of labor laws. The contractor is considered merely an agent of
the principal employer and the latter is responsible to the employees of the labor-only
contractor as if such employees had been directly employed by the principal employer.

CABALAN (CAPANELA) V. NLRC 241 SCRA 643 (1995)


FACTS: Private respondent Fernando Sanchez filed a complaint for illegal dismissal,
nonpayment of back wages and other benefits against petitioner CAPANELA
1. Private respondent alleged that there existed an employer-employee relationship
between him and CAPANELA and that the latter exercised control as employer
over the means and methods by which the work was accomplished
2. Petitioners asserted that CAPANELA is an association composed of Negritos
who worked inside the American naval base in Subic Bay. Said association
organized the system of employment of members of this cultural community who
were accorded special treatment because of the occupancy of their ancestral
lands as part of the operational area and military facility used by the Base
authorities
3. CAPANELA alleged that neither the association nor its president was the
employer of private respondent; rather, it was the US government acting through
military base authorities
4. LA held in favor of private respondent in declaring his dismissal illegal
ISSUE: WON PETITIONER IS A LABOR-ONLY CONTRACTOR
HELD: Neither can petitioners be deemed to have been engaged in permissible job
contracting under the law, for failure to satisfy the following prescribed conditions:
1. The contractor carries on an independent business and undertakes the
contract work on his own account under his own responsibility according to
his own manner and method, free from the control and direction of his
employer or principal in all matters connected with performance of the work
except as to the results thereof; and
2. The contractor has substantial capital or investment in the form of tools,
equipment, machineries, work premises and other materials which are
necessary in the conduct of his business.
In the present case, the setup was such that CAPANELA was merely tasked with
organizing the Negritos to facilitate the orderly administration of work made available to
them at the base facilities, that is, sorting scraps for recycling. CAPANELA recorded the
attendance of its members and submitted the same to the Base authorities for the
determination of wages due them and the preparation of the payroll. Payment of wages
was coursed through CAPANELA but the funds therefor came from the coffers of the
Base. Once inside the Base, control over the means and methods of work was
exercised by the Base authorities. Accordingly, CAPANELA functioned as just an
administrator of its Negrito members employed at the Base.
From the legal standpoint, CAPANELA's activities may at most be considered akin to
that of labor-only contracting, albeit of a special or peculiar type, wherein CAPANELA,
operating like a contractor, merely acted as an agent or intermediary of the employer.

ESCASINAS V. SHANGRI-LA MACTAN G.R. NO 178827 (2009)


FACTS: Petitioners Escasinas and Songco were engaged in 1999 and 1996 by Dr
Jessica Pepito to work in her clinic at respondent Shangri-La Mactan resort, where the
latter was a retained physician
1. Petitioners filed a complaint for regularization, underpayment of wages,
nonpayment of holiday pay, night shift differential and 13 th month pay differential
against respondents, claiming that they are regular employees of Shangri-La
2. Shangri-La claimed, however, that petitioners are not its employees but of the
respondent doctor via MOA pursuant to Art 157 Labor Code
3. Respondent doctor, on the other hand, claimed that petitioners were already
working for the previous retained physicians of Shangri-La before she was
retained by the resort and that she maintained petitioners services upon their
request
4. LA declared petitioners to be regular employees of Shangr-La as they usually
perform work which is necessary and desirable to the resort and that 4 elements
which determine employment relationship are present
5. NLRC reversed the LA decision and dismissed the petitioners complaint for lack
of merit, finding that there was no employer-employee relationship between
petitioner and Shangri-La. CA affirmed the same
6. Petitioners insisted that under Art 157 Labor Code, Shangri-La is required to hire
a full-time registered nurse, apart from a physician, hence, their engagement
should be deemed as regular employment and that MOA signed by Pepito and
Shangri-La is contrary to public policy as it circumvents tenurial security. At most,
the MOA is a mere job contract
7. Petitioners further averred that respondent doctor is a labor-only contractor for
she has no license or business permit which is contrary to the requirements
under Sec 19-20 of IRR of the Labor Code on subcontracting. Petitioners also
contended that respondent doctor cannot be a legitimate independent contractor
as she lacks the substantial capital and that she has no control over how the
clinic is being run
ISSUE: WON RESPONDENT DOCTOR IS A LABOR-ONLY CONTRACTOR
HELD: The existence of an independent and permissible contractor relationship is
generally established by considering the following determinants: whether the contractor
is carrying on an independent business; the nature and extent of the work; the skill
required; the term and duration of the relationship; the right to assign the performance
of a specified piece of work; the control and supervision of the work to another; the
employer's power with respect to the hiring, firing and payment of the contractor's
workers; the control of the premises; the duty to supply the premises, tools, appliances,
materials and labor; and the mode, manner and terms of payment.
On the other hand, existence of an employer- employee relationship is established
by the presence of the following determinants: (1) the selection and engagement of
the workers; (2) power of dismissal; (3) the payment of wages by whatever means; and

(4) the power to control the worker's conduct, with the latter assuming primacy in the
overall consideration.
Against the above-listed determinants, the respondent doctor is a legitimate
independent contractor. That Shangri-la provides the clinic premises and medical
supplies for use of its employees and guests, does not necessarily prove that
respondent doctor lacks substantial capital and investment. Besides, the maintenance
of a clinic and provision of medical services to its employees is required under Art 157,
are not directly related to Shangri-las principal businessoperation of hotels and
restaurants.
Since Shangri-la does not control how the work should be performed by petitioners, it is
not petitioners employer.

BABAS V. LORENZO SHIPPING CORP G.R. NO 186091 (2010)


FACTS: Respondent Lorenzo Shipping Corp (LSC) entered into an agreement with
Best Manpower Services Inc (BMSI) to provide maintenance and repair services to
LSCs container vans, heavy equipment, trailer chassis, etc. BMSI further undertook to
provide checkers to inspect all containers received for loading to and/or unloading from
its vessels
1. Simultaneous with the agreement, LSC leased its equipment, tools and tractors
to BMSI, said lease was coterminous with the agreement
2. BMSI, then, hired petitioners to work at LSC as checkers, welders, utility men etc
3. Subsequently, petitioners filed a complaint for regularization against LSC and
BMSI. But when LSC terminated its agreement with BMSI, petitioners lost their
employment
4. BMSI asserted that it is an independent contractor. It averred that it was willing to
regularize petitioners; however, some of them lacked the requisite qualifications
for the job. BMSI was willing to reassign petitioners who were willing to accept
reassignment. BMSI denied petitioners claim for underpayment of wages and
non-payment of 13th month pay and other benefits.
5. LSC, on the other hand, averred that petitioners were employees of BMSI and
were assigned to LSC by virtue of the Agreement. BMSI is an independent job
contractor with substantial capital or investment in the form of tools, equipment,
and machinery necessary in the conduct of its business. The Agreement
between LSC and BMSI constituted legitimate job contracting. Thus, petitioners
were employees of BMSI and not of LSC.
6. LA dismissed petitioners complaint citing that the latter were employees of BMSI
7. NLRC reversed the decision on the ground that based on the agreement, BMSI
was not engaged in legitimate job contracting but was a labor-only contractor
8. CA ruled that BMSI was an independent job contractor based on the provisions
of the agreement
ISSUE: WON BMSI IS ENAGAGED IN LABOR-ONLY CONTRACTING
HELD: Thus, in distinguishing between prohibited labor-only contracting and
permissible job contracting, the totality of the facts and the surrounding circumstances
of the case are to be considered.
Labor-only contracting, a prohibited act, is an arrangement where the contractor or
subcontractor merely recruits, supplies, or places workers to perform a job, work, or
service for a principal. In labor-only contracting, the following elements are present: (a)
the contractor or subcontractor does not have substantial capital or investment to
actually perform the job, work, or service under its own account and responsibility; and
(b) the employees recruited, supplied, or placed by such contractor or subcontractor
perform activities which are directly related to the main business of the principal.
On the other hand, permissible job contracting or subcontracting refers to an
arrangement whereby a principal agrees to put out or farm out with the contractor or

subcontractor the performance or completion of a specific job, work, or service within a


definite or predetermined period, regardless of whether such job, work, or service is to
be performed or completed within or outside the premises of the principal.
A person is considered engaged in legitimate job contracting or subcontracting if the
following conditions concur:
(a) The contractor carries on a distinct and independent business and undertakes
the contract work on his account under his own responsibility according to his
own manner and method, free from the control and direction of his employer or
principal in all matters connected with the performance of his work except as to
the results thereof;
(b) The contractor has substantial capital or investment; and
(c) The agreement between the principal and the contractor or subcontractor
assures the contractual employees' entitlement to all labor and occupational
safety and health standards, free exercise of the right to self-organization,
security of tenure, and social welfare benefits.
Given the above standards, we sustain the petitioners contention that BMSI is engaged
in labor-only contracting.

JOSE SONZA V. ABS-CBN G.R. NO 138051 (2004)


FACTS: ABS-CBN signed an agreement with Mel and Jay Management and Devt Corp
(MJMDC) that petitioner Sonza would co-host for Mel & Jay (radio and TV program).
ABS-CBN agreed to pay for Sonzas services a monthly talent fee
1. Subsequently, Sonza resigned as co-host of the said programs. He then filed a
complaint against ABS-CBN for nonpayment of salary, separation pay, service
incentive leave pay, 13th month pay, signing bonus, travel allowance and amount
due under Employees Stock Option Plan (ESOP)
2. ABS-CBN filed a motion to dismiss on the ground that no employer-employee
relationship existed between the parties
3. LA dismissed the complaint for lack of jurisdiction, finding that unlike an ordinary
employee, Sonza was free to perform the services he undertook to render in
accordance with his own style. NLRC and CA affirmed the same
ISSUE: WON SONZA IS AN EMPLOYEE OF RESPONDENT ABS-CBN
HELD: Independent contractors often present themselves to possess unique skills,
expertise or talent to distinguish them from ordinary employees. The specific selection
and hiring of Sonza, because of his unique skills, talent and celebrity status not
possessed by ordinary employees, is a circumstance indicative, but not conclusive,
of an independent contractual relationship. If Sonza did not possess such unique skills,
talent and celebrity status, ABS-CBN would not have entered into the Agreement with
Sonza but would have hired him through its personnel department just like any other
employee.
In any event, the method of selecting and engaging Sonza does not conclusively
determine his status. We must consider all the circumstances of the relationship, with
the control test being the most important element. Moreover, during the life of the
Agreement, ABS-CBN agreed to pay Sonzas talent fees as long as "AGENT and Jay
Sonza shall faithfully and completely perform each condition of this Agreement." Even if
it suffered severe business losses, ABS-CBN could not retrench Sonza because ABSCBN remained obligated to pay Sonzas talent fees during the life of the Agreement.
This circumstance indicates an independent contractual relationship between SONZA
and ABS-CBN.
Applying the control test to the present case, Sonza is not an employee but an
independent contractor. The control test is the most important test our courts apply in
distinguishing an employee from an independent contractor. This test is based on the
extent of control the hirer exercises over a worker. The greater the supervision and
control the hirer exercises, the more likely the worker is deemed an employee. The
converse holds true as well the less control the hirer exercises, the more likely the
worker is considered an independent contractor.
Even an independent contractor can validly provide his services exclusively to the hiring
party. In the broadcast industry, exclusivity is not necessarily the same as control. The

hiring of exclusive talents is a widespread and accepted practice in the entertainment


industry. This practice is not designed to control the means and methods of work of the
talent, but simply to protect the investment of the broadcast station. The broadcast
station normally spends substantial amounts of money, time and effort "in building up its
talents as well as the programs they appear in and thus expects that said talents remain
exclusive with the station for a commensurate period of time." Normally, a much higher
fee is paid to talents who agree to work exclusively for a particular radio or television
station. In short, the huge talent fees partially compensates for exclusivity, as in the
present case.
In a labor-only contract, there are three parties involved: (1) the "labor-only" contractor;
(2) the employee who is ostensibly under the employ of the "labor-only" contractor; and
(3) the principal who is deemed the real employer. Under this scheme, the "labor-only"
contractor is the agent of the principal. The law makes the principal responsible to
the employees of the "labor-only contractor" as if the principal itself directly hired or
employed the employees. These circumstances are not present in this case. As Sonza
admits, MJMDC is a management company devoted exclusively to managing the
careers of Sonza and his broadcast partner, Tiangco. MJMDC is not engaged in any
other business, not even job contracting. MJMDC does not have any other function
apart from acting as agent of Sonza or Tiangco to promote their careers in the
broadcast and television industry.

INSULAR LIFE V. NLRC 179 SCRA 459 (1989)


FACTS: Insular Life Assurance Co., Ltd. and Melecio T. Basiao entered into a contract,
stipulating that Basiao (1) would be an authorized agent of the Company; (2) to receive
compensation in the form of commissions; and (3) the Company's Rate Book and its
Agent's Manual, as well as all its circulars will be deemed part of the contract.
1. The contract also expressly stipulated that there was no employer-employee
relationship between the parties
2. Subsequently, the parties entered into another contract; in line with this, Basiao
organized an agency while concurrently fulfilling his commitments under the first
contract with the company. However it was terminated. As such, Basiao soguht
for a reconsideration and later sued the company in a civil action.
3. Thereafter, Basiao filed a complaint against the company and its president. The
company disputed the Ministry's jurisdiction over Basiao's claim and asserted
that he was not a company's employee, but an independent contractor. The
company had no obligation to him for unpaid commissions under the terms and
conditions of his contract.
4. LA held in favor of Basiao finding that an employer-employee relationship existed
between him and Insular Life. NLRC affirmed the same
5. Insular averred that the terms of the contract the company and Basiao entered
into expressly conferred to Basiao an independent contractor status. The
company did not have a hand in determining the time, place and means of
soliciting insurance and have set no accomplishment quotas. The company only
compensated him on the basis of results obtained. Further, it contends that they
do not constitute the decisive determinant of the nature of his engagement,
invoking precedents to the effect that the critical feature distinguishing the
status of an employee from that of an independent contractor is control,
that is, whether or not the party who engages the services of another has the
power to control the latter's conduct in rendering such services.
ISSUES: WON BASIAO IS AN EMPLOYEE OF PETITIONER COMPANY
HELD: No. The provisions of the contract which obliged Basiao to "observe and
conform to all rules and regulations which the company may from time to time
prescribe," as well as to the fact that the company prescribed the qualifications of
applicants for insurance, processed their applications and determined the amounts of
insurance cover to be issued as indicative of the control, which made Basiao, in legal
contemplation, an employee of the company.
However, in Viana vs. Alejo Al-Lagadan, it was held that that not every form of control
that the hiring party reserves to himself over the conduct of the party hired in
relation to the services rendered may be accorded the effect of establishing an
employer-employee relationship between them in the legal or technical sense of the
term. Realistically, it would be a rare contract of service that gives untrammelled
freedom to the party hired and eschews any intervention whatsoever in his performance
of the engagement.

Logically, the line should be drawn between rules that merely serve as guidelines
towards the achievement of the mutually desired result without dictating the means or
methods to be employed in attaining it, and those that control or fix the methodology
and bind or restrict the party hired to the use of such means. The first, which aim only
to promote the result, create no employer-employee relationship unlike the
second, which address both the result and the means used to achieve it. The
distinction acquires particular relevance in the case of an enterprise affected with public
interest, as is the business of insurance, and is on that account subject to regulation by
the State with respect, not only to the relations between insurer and insured but also to
the internal affairs of the insurance company.
The Insurance Code provides the rules and regulations governing the conduct of the
business and these are enforced by the Insurance Commissioner. It is expected for an
insurance company to promulgate a set of rules to guide its commission agents in
selling its policies that are in line with the law. The character of such rules (e.g. which
prescribe the qualifications of persons who may be insured, determination of the
premiums to be paid and the schedules of payment) does not invade the agent's
contractual prerogative to adopt his own selling methods or to sell insurance at
his own time and convenience, hence cannot justifiably be said to establish an
employer-employee relationship between him and the company.
The company has limited themselves to pointing out that Basiao's contract with the
company bound him to observe and conform to such rules and regulations as the latter
might from time to time prescribe. There has been no showing that any such rules or
regulations were in fact promulgated, much less that any rules existed or were
issued which effectively controlled or restricted his choice of methods or the
methods themselves of selling insurance. Absent such showing, the Court will not
speculate that any exceptions or qualifications were imposed on the express provision
of the contract leaving Basiao "... free to exercise his own judgment as to the time, place
and means of soliciting insurance."
Under the contract invoked by him, Basiao was not an employee of the petitioner, but a
commission agent, an independent contractor whose claim for unpaid commissions
should have been litigated in an ordinary civil action.

MAFINCO TRADING CORP V. OPLE 70 SCRA 139 (1976)


FACTS: Cosmos Aerated Water Factory Inc (Cosmos) appointed Mafinco as its sole
distributor of Cosmos soft drinks in Manila. Subsequently, Private respondent
Repomanta and Mafinco entered into a peddling contract whereby Repomanta agreed
to buy and sell cosmos soft drinks. The contracts were to remain in force for one year
unless terminated by either party upon five days notice to the other
1. Thereafter, Mafinco terminated the contracted with Repomanta. Repomanta,
then, filed a complaint charging the GM of Mafinco of having violated PD 21
which created the NLRC
2. Mafinco filed a motion to dismiss on the ground that NLRC had no jurisdiction
because private respondents were not its employees but were independent
contractor. It also argued that there was no termination of employee, but a
termination of contract
3. NLRC ruled in favor of Mafinco citing that private respondents were independent
contractors. DOLE secretary Ople reversed the same, finding that the
complainants were driver-salesmen of the company and that they were not
independent contractors because they had no capital of their own
ISSUE: WON PRIVATE RESPONDENTS WERE EMPLOYEES OF MAFINCO UNDER
THE PEDDLING CONTRACT
HELD: Under their peddling contracts, Repomanta and Moralde were not employees of
Mafinco but were independent contractors as found by the NLRC and its fact-finder and
by the committee appointed by the Secretary of Labor to look into the status of Cosmos
and Mafinco peddlers. They were distributors of Cosmos soft drinks with their own
capital and employees. Ordinarily, an employee or a mere peddler does not execute a
formal contract of employment. He is simply hired and he works under the direction and
control of the employer.
On the other hand, an independent contractor is "one who exercises independent
employment and contracts to do a piece of work according to his own methods and
without being subject to control of his employer except as to the result of the work."
Among the factors to be considered are whether the contractor is carrying on an
independent business; whether the work is part of the employer's general business; the
nature and extent of the work; the skill required; the term and duration of the
relationship; the right to assign the performance of the work to another; the power to
terminate the relationship; the existence of a contract for the performance of a specified
piece of work; the control and supervision of the work; the employer's powers and duties
with respect to the hiring, firing, and payment of the contractor's servants; the control of
the premises; the duty to supply the premises, tools, appliances, material and labor; and
the mode, manner, and terms of payment.
Those tests to determine the existence of an employer-employee relationship or
whether the person doing a particular work for another is an independent contractor
cannot be satisfactorily applied in the instant case. It should be obvious by now that the

instant case is a penumbral, sui generis case lying on the shadowy borderline that
separates an employee from an independent contractor.
In determining whether the relationship is that of employer and employee or whether
one is an independent contractor, "each case must be determined on its own facts and
all the features of the relationship are to be considered." On the basis of the peddling
contract, no employer-employee relationship was created. Hence, the old NLRC had no
jurisdiction over the termination of the peddling contract.

FELIX V. BUENASEDA 240 SCRA 139 (1995)


FACTS: Petitioner Dr. Felix joined the National Center for Mental Health (NCMH) as a
resident physician and after 3 years, he was promoted to Senior Resident Physician.
The Ministry of Health reorganized the NCMH pursuant to E.O. 119.
1. Under the said reorganization, Felix was appointed to the position of Sr. Resident
Physician in a temporary capacity.
2. Felix was later promoted to the position of Medical Specialist 1 (Temporary
Status) which was renewed the following year.
3. In 1988, the DOH issued Dept. Order 347, which required board certification as a
prerequisite for renewal of specialist positions in various med. centers, hospitals
and agencies.
4. Then Sec. of Health issued D.O. 478 (amending Sec.4 of D.O. 347) which
provided for an extension of appointments of Medical Specialists in cases where
termination of those who failed to meet the requirement for board certification
might result in disruption of hospital services.
5. In 1991, after reviewing petitioners service record and performance, the Medical
Credentials Committee of the NCMH recommended non-renewal of his
appointment as Medical Specialist 1.
6. He was, however, allowed to continue in the service, and receive his salary even
after being informed of the termination of his appointment.
7. The Chiefs of Service held an emergency meeting to discuss the petitioners
case. In the meeting, the overall consensus among the dept. heads was for
petitioners non-renewal where his poor performance, frequent tardiness and
inflexibility were pointed as among the factors responsible for the
recommendation not to renew his appointment.
8. The matter was referred to the CSC, which ruled that the temporary appointment
can be terminated any time and that any renewal of such appointment is within
the discretion of the appointing authority.
9. Consequently, petitioner was advised by hospital authorities to vacate his
cottage. Refusing to comply, petitioner filed a petition with the Merit System
Protection Board (MSPB) complaining about the alleged non-renewal of his
appointment, the MSPB, however, dismissed his complaint for lack of merit.
10. This decision was appealed to the Civil Service Commission (CSC) which
dismissed the same.
ISSUE: WON PETITIONER IS AN EMPLOYEE OF RESPONDENT HOSPITAL
HELD: A residency or resident physician position in a medical specialty is never a
permanent one. Residency connotes training and temporary status. It is the step taken
by a physician right after post-graduate internship (and after hurdling the Medical
Licensure Examinations) prior to his recognition as a specialist or sub-specialist in a
given field.
A physician who desires to specialize in Cardiology takes a required three-year
accredited residency in Internal Medicine (four years in DOH hospitals) and moves on to

a two or three-year fellowship or residency in Cardiology before he is allowed to take


the specialty examinations given by the appropriate accrediting college. In a similar
manner, the accredited Psychiatrist goes through the same stepladder process which
culminates in his recognition as a fellow or diplomate (or both) of the Psychiatry
Specialty Board. 16 This upward movement from residency to specialist rank,
institutionalized in the residency training process, guarantees minimum
standards and skills and ensures that the physician claiming to be a specialist
will not be set loose on the community without the basic knowledge and skills of
his specialty. Because acceptance and promotion requirements are stringent,
competitive, and based on merit, acceptance to a first year residency program is
no guaranty that the physician will complete the program. Attribution rates are high.
Some programs are pyramidal. Promotion to the next post-graduate year is based on
merit and performance determined by periodic evaluations and examinations of
knowledge, skills and bedside manner. Under this system, residents, specialty those
in university teaching hospitals enjoy their right to security of tenure only to the
extent that they periodically make the grade, making the situation quite unique as
far as physicians undergoing post-graduate residencies and fellowships are
concerned. While physicians (or consultants) of specialist rank are not subject to
the same stringent evaluation procedures, specialty societies require continuing
education as a requirement for accreditation for good standing, in addition to
peer review processes based on performance, mortality and morbidity audits,
feedback from residents, interns and medical students and research output. The
nature of the contracts of resident physicians meet traditional tests for
determining employer-employee relationships, but because the focus of
residency is training, they are neither here nor there. Moreover, stringent standards
and requirements for renewal of specialist-rank positions or for promotion to the next
post-graduate residency year are necessary because lives are ultimately at stake.

VINOYA V. NLRC 324 SCRA 469 (2000)


FACTS: Petitioner Vinoya, the complainant, worked with RFC as sales representative
until his services were terminated in 1991
1. Vinoya claimed he applied and was accepted by RFC. During his employ, he was
assigned to various supermarkets and groceries where he booked sales orders
and collected payments for RFC. After some time, he was transferred by RFC to
Peninsula Manpower Co Inc (PMCI), an agency which provides RFC with
additional contractual workers pursuant to a contract for the supply of manpower
services
2. After his transfer to PMCI, petitioner was allegedly reassigned to RFC as sales
representative. Subsequently, he was informed by RFC that his services were
terminated due to the expiration of contract of service between RFC and PMCI
3. Private respondent RFC argued that no employer-employee relationship existed
between the two parties as the petitioner was actually an employee of PMCI,
allegedly an independent contractor which has a contract of service with RFC.
RFC denied that petitioner was ever employed by RFC. RFC further contended
that the termination of its relationship with petitioner was brought about by the
expiration of the contract of service between itself and PMCI.
4. LA held in favor of petitioner on the grounds that petitioner was originally with
RFC and was merely transferred to PMCI, RFC had direct control and
supervision over petitioner, it paid Vinoyas wages and; petitioner was terminated
as per instruction of RFC
5. NLRC reversed the LA decision and opined that PMCI was an independent
contractor because it had substantial capital and as such, is the true employer of
petitioner
ISSUE: WON PETITIONER WAS AN EMPLOYEE OF RFC OR PMCI
HELD: Labor-only contracting, a prohibited act, is an arrangement where the contractor
or subcontractor merely recruits, supplies or places workers to perform a job, work or
service for a principal. In labor-only contracting, the following elements are present:
(a) The contractor or subcontractor does not have substantial capital or
investment to actually perform the job, work or service under its own
account and responsibility;
(b) The employees recruited, supplied or placed by such contractor or
subcontractor are performing activities which are directly related to the
main business of the principal.
On the other hand, permissible job contracting or subcontracting refers to an
arrangement whereby a principal agrees to put out or farm out with a contractor or
subcontractor the performance or completion of a specific job, work or service within a
definite or predetermined period, regardless of whether such job, work or service is to
be performed or completed within or outside the premises of the principal. A person is

considered engaged in legitimate job contracting or subcontracting if the following


conditions concur:
(a) The contractor or subcontractor carries on a distinct and independent
business and undertakes to perform the job, work or service on its own
account and under its own responsibility according to its own manner
and method, and free from the control and direction of the principal in
all matters connected with the performance of the work except as to
the results thereof;
(b) The contractor or subcontractor has substantial capital or investment;
and
(c) The agreement between the principal and contractor or subcontractor
assures the contractual employees entitlement to all labor and
occupational safety and health standards, free exercise of the right to
self-organization, security of tenure, and social and welfare benefits.
Previously, it was held that in order to be considered as a job contractor it is enough
that a contractor has substantial capital. In other words, once substantial capital is
established it is no longer necessary for the contractor to show evidence that it has
investment in the form of tools, equipment, machineries, work premises, among others.
The rational for this is that Article 106 of the Labor Code does not require that the
contractor possess both substantial capital and investment in the form of tools,
equipment, machineries, work premises, among others.
It may be inferred that it is not enough to show substantial capitalization or investment in
the form of tools, equipment, machineries and work premises, among others, to be
considered as an independent contractor. In fact, jurisprudential holdings are to the
effect that in determining the existence of an independent contractor relationship,
several factors might be considered such as, but not necessarily confined to, whether
the contractor is carrying on an independent business; the nature and extent of the
work; the skill required; the term and duration of the relationship; the right to assign the
performance of specified pieces of work; the control and supervision of the workers; the
power of the employer with respect to the hiring, firing and payment of the workers of
the contractor; the control of the premises; the duty to supply premises, tools,
appliances, materials and labor; and the mode, manner and terms of payment.
Based on the foregoing, PMCI can only be classified as a labor-only contractor and, as
such, cannot be considered as the employer of petitioner.

NEW GOLDEN BUILDERS & DEVT CORP V. CA 418 SCRA 411 (2003)
FACTS: Petitioner New Golden Builders entered into a construction contract with Prince
David Devt Copt for the construction of a residential condominium building. Petitioner,
then contracted Nilo Layno Builders to do the concrete works. Pursuant to the contract,
Nilo Layno Builders hired private respondents to perform work at the project
1. Private respondents filed a complaint against petitioner for unfair labor practice,
nonpayment of 13th month pay, nonpayment of 5 days service incentive leave
pay, illegal dismissal and severance pay
2. LA held that Nilo Layno Builders was a labor-only contractor, thus private
respondents were deemed employees of petitioner
3. Petitioner claimed that Nilo Layno Builders was an independent contractor
4. NLRC affirmed the same but added that private respondents were illegally
dismissed by petitioner
ISSUE: WON NILO LAYNO BUILDERS WAS AN INDEPENDENT CONTRACTOR
AND WON THERE EXISTED AN EMPLOYER-EMPLOYEE RELATIONSHIP
BETWEEN PETITIONER AND PRIVATE RESPONDENTS
HELD: Under Section 8, Rule VIII, Book III, of the Omnibus Rules Implementing the
Labor Code, an independent contractor is one who undertakes job contracting, i.e., a
person who: (a) carries on an independent business and undertakes the contract work
on his own account under his own responsibility according to his own manner and
method, free from the control and direction of his employer or principal in all matters
connected with the performance of the work except as to the results thereof; and (b) has
substantial capital or investment in the form of tools, equipments, machineries, work
premises, and other materials which are necessary in the conduct of the business.
Jurisprudential holdings are to the effect that in determining the existence of an
independent contractor relationship, several factors may be considered, such as, but
not necessarily confined to, whether or not the contractor is carrying on an independent
business; the nature and extent of the work; the skill required; the term and duration of
the relationship; the right to assign the performance of specified pieces of work; the
control and supervision of the work to another; the employers power with respect to the
hiring, firing and payment of the contractors workers; the control of the premises; the
duty to supply premises, tools, appliances, materials and labor; and the mode, manner
and terms of payment.
The test to determine the existence of independent contractorship is whether one
claiming to be an independent contractor has contracted to do the work according to his
own methods and without being subject to the control of the employer, except only to
the results of the work.
This is exactly the situation obtaining in the case at bar. Nilo Layno Builders hired its
own employees, the private respondents, to do specialized work in the Prince David
Project of the petitioner. The means and methods adopted by the private respondents
were directed by Nilo Layno Builders except that, from time to time, the engineers of the

petitioner visited the site to check whether the work was in accord with the plans and
specifications of the principal. As admitted by Nilo G. Layno, he undertook the contract
work on his own account and responsibility, free from interference from any other
persons, except as to the results; that he was the one paying the salaries of private
respondents; and that as employer of the private respondents, he had the power to
terminate or dismiss them for just and valid cause. Indubitably, the Court finds that Nilo
Layno Builders maintained effective supervision and control over the private
complainants.
In legitimate job contracting, the law creates an employer-employee relationship for a
limited purpose, i.e., to ensure that the employees are paid their wages. The principal
employer becomes jointly and severally liable with the job contractor only for the
payment of the employees wages whenever the contractor fails to pay the same. Other
than that, the principal employer is not responsible for any claim made by the
employees.
The joint and several liability of the employer or principal was enacted to ensure
compliance with the provisions of the Code, principally those on statutory minimum
wage. The contractor or subcontractor is made liable by virtue of his or her status as a
direct employer, and the principal as the indirect employer of the contractors
employees. This liability facilitates, if not guarantees, payment of the workers
compensation, thus, giving the workers ample protection as mandated by the 1987
Constitution. This is not unduly burdensome to the employer. Should the indirect
employer be constrained to pay the workers, it can recover whatever amount it had paid
in accordance with the terms of the service contract between itself and the contractor.
This liability covers the payment of service incentive leave and 13 th month pay of the
private complainants during the time they were working at petitioners Prince David
Project. So long as the work, task, job or project has been performed for petitioners
benefit or on its behalf, the liability accrues for such period even if, later on, the
employees are eventually transferred or reassigned elsewhere.

SAN MIGUEL CORP V. MAERC INTEGRATED SYSTEMS 405 SCRA 579 (2003)
FACTS: 291workers filed their complaints against San Miguel Corporation (SMC) and
Maerc Integrated Services, Inc., for illegal dismissal, underpayment of wages etc.
1. The complainants alleged that they were hired by SMC through its agent MAERC
to work inside the SMC premises and in the Philphos Warehouse owned by
MAERC.
2. They washed and segregated various kinds of empty bottles used by SMC to sell
and distribute its beer beverages to the consuming public.
3. They were paid on a per piece or pakiao basis except for a few who worked as
checkers and were paid on daily wage basis.
4. SMC cited its plans to phase out its segregation activities due to the installation
of labor and cost saving devices.
5. When the service contract was terminated, respondents filed a complaint for
illegal dismissal. .
6. The LA rendered a decision holding that MAERC was an independent contractor.
7. On appeal, the NLRC ruled that MAERC was a labor-only contractor and that
complainants were employees of SMC. The NLRC also held that whether
MAERC was a job contractor or a labor-only contractor, SMC was still solidarily
liable with MAERC for the latter's unpaid obligations.
8. The CA affirmed the decision of the NLRC.
ISSUE: WON SMC IS SOLIDARILY LIABLE WITH MAERC
HELD: In legitimate job contracting, the law creates an employer-employee relationship
for a limited purpose, i.e., to ensure that the employees are paid their wages. The
principal employer becomes jointly and severally liable with the job contractor only for
the payment of the employees' wages whenever the contractor fails to pay the same.
Other than that, the principal employer is not responsible for any claim made by the
employees.
On the other hand, in labor-only contracting, the statute creates an employer-employee
relationship for a comprehensive purpose: to prevent a circumvention of labor laws. The
contractor is considered merely an agent of the principal employer and the latter is
responsible to the employees of the labor-only contractor as if such employees had
been directly employed by the principal employer. The principal employer therefore
becomes solidarily liable with the labor-only contractor for all the rightful claims of the
employees.
This distinction between job contractor and labor-only contractor, however, will not
discharge SMC from paying the separation benefits of the workers, inasmuch as
MAERC was shown to be a labor-only contractor; in which case, petitioner's liability is
that of a direct employer and thus solidarily liable with MAERC.

COCA-COLA BOTTLERS V. DELA CRUZ G.R. NO 184977 (2009)


FACTS: Respondents Dela Cruz filed a case for regularization with money claims
against petitioner Coca-Cola Bottlers Inc
1. Respondents alleged that they were route helpers assigned to work in
petitioners trucks. They go from the Coca-Cola sales offices or plants to
customer outlets; they were hired either directly by petitioner or by its contractors
but they do not enjoy full remuneration, benefits and privileges granted to
petitioners regular sales force. They argued that services they render are
necessary and desirable in the regular business of petitioner
2. Petitioner contended that it had entered into contracts of services with Peerless
and Excellent Partners Cooperative Inc (Excellent) to provide allied services;
under these contracts, Peerless and Excellent retained the right to select, hire,
dismiss, supervise, control and discipline and pay the salaries of all personnel
assigned to petitioner. In turn, both companies were paid a stipulated fee.
Petitioner posited that no employer-employee relationship existed between
respondents and petitioner
3. LA dismissed the case for lack of jurisdiction, citing that respondents were not
employees of petitioner as they were employees of either Peerless or Excellent.
NLRC affirmed the same
4. CA reversed the LA ruling citing that based on the contractual agreements
between Peerless and Excellent, on one hand, and the company, on the other,
the former were engaged in labor-only contracting. CA ruled that the defect in the
verification and certification was a mere formal requirement that can be excused
in the interest of substantial justice, following the ruling of this Court in Uy v.
Landbank of the Philippines.
ISSUE: WHAT IS THE EFFECT OF THE LACK OF DOLE CERTIFICATION OF
PEERLESS AND EXCELLENT AS LEGITIMATE JOB CONTRACTOR
HELD: The petitioners belated attention to the imputed defect indicates that the
petitioner did not consider this defect worth raising when things were going its way, but
considered it a serious one when things turned the other way. This opportunistic stance
is not our idea of how technical deficiencies should be viewed. Under the circumstances
of this case, the defect is a technical and minor one; the respondents did file the
required verification and certification of non-forum shopping with all the respondents
properly participating, marred only by a glitch in the evidence of their identity. In the
interest of justice, this minor defect should not defeat their petition and is one
that we can overlook in the interest of substantial justice, taking into account the
merits of the case.

MANILA PUBLIC SCHOOL TEACHERS ASSOC V. LAGUIO 200 SCRA 323 (1991)
FACTS: This involves 2 consolidated cases about a series of events that started with
the so-called mass action undertaken by some 800 public school teachers, among
them members of the petitioning associations.
1. Petitioners alleged that they resolved to engage in mass concerted actions, after
peaceful dialogues with the heads of the Department of Budget and
Management, HOR and Senate, as well as after exhausting all administrative
remedies for the immediate payment of due chalk, clothing allowances, 13 th
month pay arising from the salary standardization law, etc.
2. On September 14, 1990, petitioners and teachers in other cities and
municipalities in Metro Manila, staged a protest rally at the DECS premises
without disrupting classes as a last call for the government to negotiate the
granting of demands. No response was made by DECS secretary despite the
demonstration so the petitioners began the ongoing protest mass actions on
September 17, 1990
3. However, they were issued a return to work order, ordering them to report for
work within 24 hours or face dismissal, and a memorandum directing DECS
officials concerned to initiate dismissal proceedings against those who did not
comply and hire their replacements. Despite this, the mass actions continued into
the week, with more teachers joining in the days that followed
4. Based on the record of the principals from various public schools in Metro
Manila, DECS secretary filed motu proprio administrative complaints against the
teachers who had taken part in the mass actions and defied the return to work
order on assorted charges like grave misconduct, gross neglect of duty, gross
violation of Civil Service Law, absence without official leave, etc. Subsequently,
DECS secretary constituted an investigating committee to determine and take
the appropriate course of action
5. Consequently, DECS secretary found the 20 teachers guilty of the charges
preferred against them and dismissed them from office effective immediately. In
other investigations that followed, 658 teachers were dismissed, others were
suspended
6. RTC held in favor of DECS secretary, citing that employees in the public (civil)
service, unlike those in the private sector, do not have the right to strike, although
guaranteed the right to self-organization, to petition Congress for the betterment
of employment terms and conditions and to negotiate with appropriate
government agencies for the improvement of such working conditions as are not
fixed by law
ISSUE: WON PETITIONERS HAVE A RIGHT TO STRIKE
HELD: Petitioners, who are public schoolteachers and thus government employees, do
not seek to establish that they have a right to strike. Rather, they tenaciously insist that
their absences during certain dates in September 1990 were a valid exercise of their
constitutional right to engage in peaceful assembly to petition the government for a
redress of grievances. They claim that their gathering was not a strike; therefore, their

participation therein did not constitute any offense. These 'mass actions' were to all
intents and purposes a strike; they constituted a concerted and unauthorized stoppage
of, or absence from, work which it was the teachers' duty to perform, undertaken for
essentially economic reasons," should not principally resolve the present case, as the
underlying facts are allegedly not identical.
It has long been settled that the mass actions of September/October 1990 staged by
Metro Manila public school teachers amounted to a strike in every sense of the term,
constituting, as they did, "concerted and unauthorized stoppage of or absence from,
work which it was the teachers" duty to perform, undertaken for essentially economic
reasons." The claim that the teachers involved in the 1990 mass actions were merely
exercising their constitutional right to peaceful assembly was already rejected in Gan vs.
Civil Service Commission.
DISSENTING:
GUTTIEREZ: Employees in the civil service may not engage in strikes, walk-outs and
temporary work stoppages like workers in the private sector. Employment in the
Government is governed by law. Government workers cannot use the same weapons
employed by workers in the private sector to secure concessions from their employers.
The terms and conditions of employment are effected through statutes and
administrative rules and regulations, not through collective bargaining agreements.
When Government consistently fails to act on these grievances, the teachers have a
right to speak in an effective manner. For speech to be effective, it must be forceful
enough to make the intended recipients listen.
To me, the issue is the freedom to effectively speak. When the members of a noble
profession are demeaned by low salaries and inattention to their needs, surely their
freedom to speak in a manner and at a time as is most effective far outweighs
conventional adherence to orthodox civil service rules on proper conduct and correct
behavior.
CRUZ: Assuming it to be correct, the prohibition on strike is no license for the
authorities to treat their employees with disdain and to flatly ignore their legitimate
complaints, with the expressed threat that they would be removed if they should be so
rash as to insist on their demands. In my view, that is what Secretary Carino has done.
Government workers, whatever their category or status, have as much right as any
person in the land to voice their protests against what they believe to be a violation of
their interests. The fact that they belong to the Civil Service has not deprived them of
their freedom of expression, which is guaranteed to every individual in this country,
including even the alien. It would be ridiculous to even suggest that by accepting public
employment, the members of the Civil Service automatically and impliedly renounce this
basic liberty. This freedom can at best be regulated only but never completely
withdrawn.

LUMANTA V. NLRC 170 SCRA 79 (1989)


FACTS: Petitioner Lumanta joined by 54 other retrenched employees, filed a compliant
for unpaid retrenchment or separation pay, underpayment of wages and nonpayment of
ECOLA against private respondent Food Terminal Inc (FTI)
1. FTI argued that it was a government-owned and controlled corporation, its
employees are governed by the Civil Service Law and not by the Labor Code,
and that claims arising from employment fall within the jurisdiction of the CSC
and not DOLE
2. Petitioner further argued that although FTI is a GOCC, it still has the marks of a
private corporation: it directly hires its employees without seeking approval from
CSC and its employees are covered by SSS and not GSIS. Petitioners also
averred that being a GOCC without original charter, FTI falls outside the scope of
CSC as marked out in Sec 2(1) Art IX of the 1987 Constitution
3. LA dismissed the complaint for lack of jurisdiction. NLRC affirmed the same
ISSUE: WON A LABOR LAW CLAIM AGAINST A GOCC, SUCH AS PRIVATE
RESPONDENT FTI, FALLS WITHIN THE JURISDICTION OF DOLE
HELD: Yes.
In NASECO v. NLRC< the SC held that GOCCs with original charter refer to
corporations chartered by special law as distinguished from corporations organized
under the Corporation Code.
It is the 1987 Constitution and not the case law embodied in NHA v. Juco, which applies
in the case at bar, under the principle that jurisdiction is determined as of the time of the
filing of the complaint. At the time the complaint was filed against FTI and at the time
the decisions of LA and NLRC were rendered, the 1987 Constitution had already come
into effect.
Based on the facts presented, FTI is a GOCC without original charter therefore it is the
DOLE and not CSC, which has jurisdiction over the dispute arising from the
employment of petitioners with private respondent FTI, and that consequently, the terms
and conditions of such employment are governed by the Labor Code and not by the
Civil Service Rules and Regulations

INTERPHIL
LABORATIES
EMPLOYEES
LABORATORIES INC 372 SCRA 658 (2001)

UNION-FFW

V.

INTERPHIL

FACTS: Interphil Laboratories Employees Union-FFW is the sole and exclusive


bargaining agent of the rank-and-file employees of respondent Interphil Laboratories
Inc, a company engaged in the business of manufacturing and packaging
pharmaceutical products. They had a CBA effective from August 1, 1990 to July 31,
1993
1. Prior to the expiration of the CBA sometime in February 1993, Salazar, the VP of
the human resources department, was approached by the union officers to
inquire about the stand of the company regarding the duration of the CBA which
was set to expire soon. Salazar informed them that it would be discussed during
the formal negotiations which would start soon
2. Salazar was again approached by the union officers regarding the same issue to
which the former gave the same response. The next day, all rank-and-file
employees refused to follow their regular 2-shift work and left their workplace
without sealing the containers and securing the raw materials they were working
on. When Salazar inquired about the reason for their refusal to follow their
normal work schedule, the employees told him to ask the union officers.
3. Salazar was then informed by the union officers that the employees would only
return to their normal work schedule if the company would agree to their
demands as to the duration and effectivity of the new CBA. Since the union was
apparently unsatisfied with the answer of the management, the overtime boycott
continued. In addition, the employees started to engage in a work slowdown
campaign during the time they were working, thus substantially delaying the
production
4. Subsequently, the petitioner union submitted its CBA and the company filed its
counter-proposal.
5. Respondent then filed with NLRC a petition to declare petitioner unions overtime
boycott and work slowdowas illegal strike
6. In February 1994, DOLE secretary issued an assumption order over the labor
dispute and ordered the respondent company to immediately accept all striking
workers under the same terms and conditions prior to the strike and for the
petitioner union to strictly and immediately company with the return to work
orders issued
7. Based on the report of the LA, DOLE declared the overtime boycott and work
slowdown as illegal strike and found the petitioner union guilty of unfair labor
practice for violating the existing CBA which prohibits the union or any employee
during the existence of the CBA from staging a strike or engaging in slowdown or
interruption of work
8. CA affirmed the same
9. Petitioner alleged that the provisions of their CBA on working hours clearly stated
that the normal working hours were from 7:30 am to 4:30 pm. Bu

ISSUE: WON PETITIONER IS CORRECT IN ALLEGING THAT RESPONDENT


COMPANY VIOLATED THE CBA WHEN IT SCHEDULED THE WORKING HOURS IN
SHIFTING SCHEDULE
HELD: Sec 1 of the CBA states:
A normal workday shall consist of not more than eight (8) hours. The
regular working hours for the Company shall be from 7:30 A.M. to 4:30
P.M. The schedule of shift work shall be maintained; however the
company may change the prevailing work time at its discretion, should
such change be necessary in the operations of the Company. All
employees shall observe such rules as have been laid down by the
company for the purpose of effecting control over working hours.
It is evident from the foregoing provision that the working hours may be changed, at the
discretion of the company, should such change be necessary for its operations, and that
the employees shall observe such rules as have been laid down by the company. It was
established that the employees adhered to the said work schedule since 1988. The
employees are deemed to have waived the eight-hour schedule since they followed,
without any question or complaint, the two-shift schedule while their CBA was still in
force and even prior thereto. The two-shift schedule effectively changed the working
hours stipulated in the CBA. As the employees assented by practice to this
arrangement, they cannot now be heard to claim that the overtime boycott is justified
because they were not obliged to work beyond eight hours.

DUNCAN ASSOC OF DETAILMAN-PGWTO AND TECSON V. GLAXO WELLCOME


PHILS G.R. NO 164774 (2006)
FACTS: Petitioner Tecson was hired by respondent Glaxo Wellcome Phils Inc., as
medical representative in October 1995.
1. The Employee Code of Conduct of Glaxo states that an employee employee is
expected to inform management of any existing or future relationship by
consanguinity or affinity with co-employees or employees of competing drug
companies. If management perceives a conflict of interest or a potential conflict
between such relationship and the employees employment with the company,
the management and the employee will explore the possibility of a "transfer to
another department in a non-counterchecking position" or preparation for
employment outside the company after six months.
2. Subsequently, Tecson entered into romantic relationship with Bettsy, an
employee of Astra Pharmaceuticals, a competitor of Glaxo. Bettsy was Astras
branch manager in Albay
3. Even before they got married, Tecson received several reminders from his
district manager regarding the conflict of interest his relationship with Bettsy
might engender. Still, Tecson married Bettsy in September 1998
4. Eventually, Tecsons superiors informed him that he and Bettsy should decide
which one of them should resign from their jobs to comply with the company
policy. Tecson requested for time to comply with the company policy.
5. In November 1999, Glaxo transferred Tecson to a different area. Tecson sough
reconsideration for his transfer and brought the matter to Glaxos grievance
committee but the latter denied his request. Tecson defied the transfer order and
continued acting as medical representative in the Camarines Sur sales area
6. Petitioner alleged that Glaxos policy against employees marrying employees of
competitor companies violates the equal protection clause of the Constitution
because it creates invalid distinctions among employees on account only of
marriage
7. Glaxo contended that the company policy prohibiting its employees from having
relationship with and/or marrying an employee of a competitor company is a valid
exercise of its management prerogatives and does not violate the equal
protection clause; and that Tecsons reassignment from the Camarines NorteCamarines Sur sales area to the Butuan City-Surigao City and Agusan del Sur
sales area does not amount to constructive dismissal
8. Glaxo insists that as a company engaged in the promotion and sale of
pharmaceutical products, it has a genuine interest in ensuring that its employees
avoid any activity, relationship or interest that may conflict with their
responsibilities to the company. Thus, it expects its employees to avoid having
personal or family interests in any competitor company which may influence their
actions and decisions and consequently deprive Glaxo of legitimate profits. The
policy is also aimed at preventing a competitor company from gaining access to
its secrets, procedures and policies.

ISSUES: WON GLAXOS POLICY AGAINST ITS EMPLOYEES MARRYING


EMPLOYEES FROM COMPETITOR COMPANIES IS VALID; WON TECSON WAS
COSNTRUCTIVELY DISMISSED
HELD: Yes, Glaxo has a right to guard its trade secrets, manufacturing formulas,
marketing strategies and other confidential programs and information from competitors,
especially so that it and Astra are rival companies in the highly competitive
pharmaceutical industry.
The prohibition against personal or marital relationships with employees of competitor
companies upon Glaxos employees is reasonable under the circumstances because
relationships of that nature might compromise the interests of the company. In laying
down the assailed company policy, Glaxo only aims to protect its interests against the
possibility that a competitor company will gain access to its secrets and procedures. It is
clear that Glaxo does not impose an absolute prohibition against relationships between
its employees and those of competitor companies. Its employees are free to cultivate
relationships with and marry persons of their own choosing. What the company merely
seeks to avoid is a conflict of interest between the employee and the company that may
arise out of such relationships.
No less than the Constitution recognizes the right of enterprises to adopt and enforce
such a policy to protect its right to reasonable returns on investments and to expansion
and growth. Indeed, while our laws endeavor to give life to the constitutional policy on
social justice and the protection of labor, it does not mean that every labor dispute will
be decided in favor of the workers. The law also recognizes that management has rights
which are also entitled to respect and enforcement in the interest of fair play.
The challenged company policy does not violate the equal protection clause of the
Constitution as petitioners erroneously suggest. It is a settled principle that the
commands of the equal protection clause are addressed only to the state or those
acting under color of its authority. The only exception occurs when the state in any of its
manifestations or actions has been found to have become entwined or involved in the
wrongful private conduct. However, the exception is not present in this case.
Significantly, the company actually enforced the policy after repeated requests to the
employee to comply with the policy. Indeed, the application of the policy was made in an
impartial and even-handed manner, with due regard for the lot of the employee.
It is likewise erroneous to conclude that Tecson was constructively dismissed when he
was transferred to Butuan City sales area. Constructive dismissal is defined as a
quitting, an involuntary resignation resorted to when continued employment becomes
impossible, unreasonable, or unlikely; when there is a demotion in rank or diminution in
pay; or when a clear discrimination, insensibility or disdain by an employer becomes
unbearable to the employee. None of these conditions are present in the instant case.
The record does not show that Tescon was demoted or unduly discriminated upon by
reason of such transfer.

SOLID DEVT CORP WORKERS ASSOC V. SOLID DEVT CORP 530 SCRA 132
(2007)
FACTS: Petitioners Villena and Colcol were employed as roving doffer and trouble
shooter mechanic, respectively, by private respondent Solid Development Corp (SDC)
1. In May 1999, Gaw, the owner and president of SDC, caught Villena loafing
during office hours and when he called Villenas attention, Villena answered in a
rude manner (Bakit mo ako sinisita porke mahirap lang kami mga trabahador
ninyo. Kayo talagang mga instik. Ikaw, masyado kang sipsip sa baboy na instik)
2. Villena then was served an infraction report charging him with disrespect to a
superior officer and/or impolite/discourteous manner. He was required to submit
a written answer within 12 hours from receipt of report but the failed to reply. He
was subsequently dismissed for serious misconduct, loss of confidence and
gross habitual neglect of duty
3. Gaw, Colcols supervisor, ordered the latter to operate the carding or rolyohan
machine. Colcol refused and explained he did not know how to operate the
machine. Colcol was issued an infraction report for insubordination and poor
work performance, and was required to submit a written explanation within 12
hours from receipt. Colcol was eventually dismissed for insubordination and poor
work performance
4. Petitioners filed separate complaints for illegal dismissal with prayer for
reinstatement and money claims, they claimed they were dismissed without just
cause and without due process
5. LA held in favor of Villena and Colcol and ordered SDC to reinstate complainants
to their former position without loss of seniority rights and other privileges with full
back wages
6. NLRC reversed the LA decision, giving more credence to private respondents
assertion that Colcol received the infraction report but simply ignored it. CA
affirmed the same
ISSUE: WON PETITIONERS VILLENA AND COLCOL WERE DISMISSED FOR
CAUSE AND DUE PROCESS
HELD: Yes. It is settled that to constitute a valid dismissal from employment, two requisites
must concur: (1) the dismissal must be for any of the causes provided for in Art 282 Labor
Code; and (2) the employee must be afforded an opportunity to be heard and to defend
himself. This means that an employer can terminate the services of an employee for just
and valid causes, which must be supported by clear and convincing evidence. It also
means that, procedurally, the employee must be given notice, with adequate opportunity to
be heard, before he is notified of his actual dismissal for cause.
For serious misconduct to be a just cause for dismissal, it must (1) be serious; (2) relate
to the performance of the employees duties; and (3) show that the employee has
become unfit to continue working for the employer. Villenas act of insulting Gaw, the
companys owner and president, may be considered, from a laymans perspective, as a

serious misconduct. Moreover, it was done in relation to the performance of his duties
as would show him to be unfit to continue working for the company.
Similarly, Colcols excuse in refusing to operate the carding or rolyohan machine was
properly rejected. First, as troubleshooter or all-around mechanic, he was tasked to
maintain and repair all of the companys equipment including the carding or rolyohan
machine. Second, the machine has been used by the company for many years.
Because of these, Colcol could not have been ignorant of its proper operation.
Willful disobedience of the employers lawful orders, as a just cause for dismissal of an
employee, envisages the concurrence of at least two requisites: (1) the employees
assailed conduct must have been willful, that is, characterized by a wrongful and
perverse attitude; and (2) the order violated must have been reasonable, lawful, made
known to the employee and must pertain to the duties which he had been engaged to
discharge.
Likewise, there was nothing unreasonable in the order. It is the employers prerogative,
based on its assessment and perception of its employees qualifications, aptitudes and
competence, to move the employee around in the various areas of its business
operations in order to ascertain where he will function with utmost efficiency and
maximum productivity or benefit to the company. An employees right to security of
tenure does not give him such a vested right in his position as would deprive the
company of its prerogative to change his assignment or transfer him where he will be
most useful. In this case, it is presumed that private respondents have carefully
evaluated Colcols competence as troubleshooter mechanic to require him to operate
the carding or rolyohan machine.

BAGUIO COUNTRY CLUB V. NLRC 206 SCRA 643 (1992)


FACTS: Private respondent Calamba was employed on a day to day basis in various
capacities as laborer and dishwasher for a period of 10 months from October 1979 to
July 1980. From September 1980 to October 1980, he was hired as a gardener and
rehired as such in November 1980 until January 1981 when he was dismissed by
petitioner Baguio Country Club
1. Private respondent then filed a complaint against petitioner for unfair labor
practice, illegal dismissal and nonpayment of 13th month pay for 1979 and 1980
2. LA held in favor of private respondent based on the following findings:
a. Calamba has attained regular status as an employee of the club on account
of the nature he was hired, to perform continuously and on staggered basis
for a span of 13 months
b. Being a regular employee with more than 1 year of service, Calamba could
not be terminated without just or valid cause
3. Petitioner filed an appeal to NLRC; NLRC dismissed the petition for lack of merit
4. Petitioner alleged that private respondent was a contractual employee whose
employment was for a fixed and specific period as set forth and evidenced by the
private respondents contract of employment
5. NLRC maintained that Calamba, having rendered services as laborer, gardener
and dishwasher for more than 1 year, was a regular employee at the time of his
dismissal. Moreover, the nature of Calambas employment as laborer, gardener
and dishwasher pertains to a regular employee because they are necessary or
desirable in the usual business of petitioner as a recreational establishment
ISSUE: WON PRIVATE RESPONDENT HAS ACQUIRED THE STATUS OF A
REGULAR EMPLOYEE AT THE TIME HIS EMPLOYMENT WAS TERMINATED
HELD: The primary standard, therefore, of determining a regular employment is the
reasonable connection between the particular activity performed by the employee in
relation to the usual business or trade of the employer. The test is whether the former is
usually necessary or desirable in the usual business or trade of the employer. The
connection can be determined by considering the nature of the work performed and its
relation to the scheme of the particular business or trade in its entirety. Also, if the
employee has been performing the job for at least one year, even if the performance is
not continuous or merely intermittent, the law deems the repeated and continuing need
for its performance as sufficient evidence of the necessity if not indispensability of that
activity to the business. Hence, the employment is also considered regular, but only with
respect to such activity and while such activity exists. In the case at bar, the records
show that the petitioner hired private respondent repeated to perform tasks ranging from
dishwashing and gardening, aside from performing maintenance work. Such repeated
rehiring and the continuing need for his service are sufficient evidence of the necessity
and indispensability of his service to the petitioners business trade.
It is of no moment that private respondent was told when he was hired that his
employment would only be "on a day to day basis for a temporary period" and may be

terminated at any time subject to the petitioner's discretion. Precisely, the law overrides
such conditions which are prejudicial to the interest of the worker. Evidently, the
employment contracts entered into by private respondent with the petitioner have the
purpose of circumventing the employee's security of tenure. The Court therefore,
rigorously disapproves said contracts which demonstrate a clear attempt to exploit the
employee and deprive him of the protection sanctioned by the Labor Code.
It is noteworthy that what determines whether a certain employment is regular or casual
is not the will and word of the employer, to which the desperate worker often accedes. It
is the nature of the activities performed in relation to the particular business or trade
considering all circumstances, and in some cases the length of time of its performance
and its continued existence.

MILLARES V. NLRC 285 SCRA 305 (2002)


FACTS: Petitioners were employed by private respondent ESSO Intl through its local
manning agency, private respondent Trans-Global Maritime Agency Inc.
1. Petitioner applied for a leave of absence for the period of July 9-August 7, 1989.
Subsequently, he applied for the optional retirement plan under the Consecutive
Enlistment Incentive Plan (CEIP), considering that he had already rendered more
than 20 years of continuous service
2. His request for optional retirement was denied by ESSO on the following
grounds:
a. that he was employed on a contractual basis
b. his contract of enlistment did not provide for retirement before the age of 60
c. he did not comply with the requirement for claiming benefits under CEIP
3. Subsequently, petitioner requested for an extension of his leave but instead, he
receive a letter that in view of his absence without leave, which was equivalent to
abandonment of his position, he had been dropped from the roster of crew
members
4. Petitioner, then, filed a complaint with POEA for illegal dismissal and
nonpayment of employee benefits against ESSO and Trans-Global
5. POEA dismissed the complaint for lack of merit. NLRC affirmed the same
6. Private respondents contend, among others, that:
a. The earlier ruling holding petitioners as regular employees was contrary to the
decision in Coyoca v. NLRC
b. Art 280 Labor Code is not applicable as what applies is the POEA rules and
regulations governing overseas employment
c. Seafarers are not regular employees based on international maritime practice
ISSUE: WON PETITIONERS ARE CONTRACTUAL EMPLOYEES WHOSE
EMPLOYMENT ARE TERMINATED EVERY TIME THEIR CONTRACTS EXPIRE
HELD: It is clear that seafarers are considered contractual employees. They cannot be
considered as regular employees under Art 280 Labor Code. Their employment is
governed by the contracts they sign every time they are rehired and their employment is
terminated when the contract expires. Their employment is contractually fixed for a
certain period of time. They fall under the exception of Art 280 whose employment has
been fixed for a specific project or undertaking the completion or termination of which
has been determined at the time of engagement of the employee or where the work or
services to be performed is seasonal in nature and the employment is for the duration of
the season.
Petitioners insist that they should be considered regular employees, since they have
rendered services which are usually necessary and desirable to the business of their
employer, and that they have rendered more than 20 years of service. While this may
be true, the Brent case has, however, held that there are certain forms of employment
which also require the performance of usual and desirable functions and which exceed
one year but do not necessarily attain regular employment status under Art 280.

Overseas workers including seafarers fall under this type of employment which are
governed by the mutual agreements of the parties.
As clearly stated in the Coyoca case, Filipino seamen are governed by the Rules and
Regulations of the POEA.
The Standard Employment Contract governing the
employment of All Filipino seamen on Board Ocean-Going Vessels of the POEA,
particularly in Part I, Sec. C specifically provides that the contract of seamen shall be for
a fixed period. And in no case should the contract of seamen be longer than 12
months. It reads:
Section C. Duration of Contract
The period of employment shall be for a fixed period but in no case to
exceed 12 months and shall be stated in the Crew Contract. Any
extension of the Contract period shall be subject to the mutual consent of
the parties.
Moreover, it is an accepted maritime industry practice that employment of seafarers are
for a fixed period only. Constrained by the nature of their employment which is quite
peculiar and unique in itself, it is for the mutual interest of both the seafarer and the
employer why the employment status must be contractual only or for a certain period of
time. Seafarers spend most of their time at sea and understandably, they can not stay
for a long and an indefinite period of time at sea. Limited access to shore society during
the employment will have an adverse impact on the seafarer. The national, cultural and
lingual diversity among the crew during the COE is a reality that necessitates the
limitation of its period.
Petitioners make much of the fact that they have been continually re-hired or their
contracts renewed before the contracts expired (which has admittedly been going on for
twenty (20) years). By such circumstance they claim to have acquired regular status
with all the rights and benefits appurtenant to it. Such contention is untenable.
Undeniably, this circumstance of continuous re-hiring was dictated by practical
considerations that experienced crew members are more preferred. Petitioners were
only given priority or preference because of their experience and qualifications but this
does not detract the fact that herein petitioners are contractual employees. They cannot
be considered regular employees.

PINES CITY EDUCATIONAL CENTER V. NLRC 227 SCRA 655 (1993)


FACTS: Private respondents were all teachers employed on probationary basis by
petitioner Pines City Education Center. All private respondents, with the exception of
Picart and Chan, signed contracts of employment with petitioner for a fixed duration.
1. Due to the expiration of the private respondents contracts and their poor
performance as teachers, they were notified of petitioners decision not to renew
their contracts anymore
2. Private respondents subsequently filed a complaint for illegal dismissal, alleging
that their dismissal was without cause and in violation of due process. Arbiter,
alleging that their dismissals were without cause and in violation of due process.
Except for private respondent Leila Dominguez who worked with petitioners for
one semester, all other private respondents were employed for one to two years.
They were never informed in writing by petitioners regarding the standards or
criteria of evaluation so as to enable them to meet the requirements for
appointment as regular employees. They were merely notified in writing by
petitioners
3. Petitioners contended that private respondents' separation from employment,
apart from their poor performance, was due to the expiration of the periods
stipulated in their respective contracts.
4. LA held in favor of private respondents citing that the teachers contracts are
vague and do not include the specific description of duties and assignments of
private respondents. Petitioners also did not present any written evidence to
support their allegation that the Academic Committee has evaluated private
respondents performance. NLRC affirmed the same

ISSUE: WON PRIVATE RESPONDENTS WERE ILLEGALLY DISMISSED (WON


THEY WERE REGULAR EMPLOYEES)
HELD: Art 280 Labor Code should have no application to instances where a fixed
period of employment was agreed upon knowingly and voluntarily by the parties, without
any force, duress or improper pressure brought to bear upon the employee and absent
any other circumstances vitiating his consent, or where it satisfactorily appears that the
employer or employee dealt with each other on more or less equal terms with no moral
dominance whatever being exercised by the former over the latter.
Insofar as the private respondents who knowingly and voluntarily agreed upon fixed
periods of employment are concerned, their services were lawfully terminated by reason
of the expiration of the periods of their respective contracts.
With respect to private respondents Roland Picart and Lucia Chan, both of whom did
not sign any contract fixing the periods of their employment nor to have knowingly and
voluntarily agreed upon fixed periods of employment, petitioners have the burden of
proving that the termination of their services was legal. As probationary employees, they
are likewise protected by the security of tenure provision of the Constitution.

Consequently, they cannot be removed from their positions unless for cause. On the
other hand, petitioner contended that based on the evaluation of the Academic
Committee their performance as teachers was poor. However, there is absolutely
nothing in the record which will show that the complainants were afforded even an iota
of chance to refute respondents' allegations that the complainants did not meet the
reasonable standards and criteria set by the school. Thus, the order for their
reinstatement and payment of full back wages and other benefits and privileges from
the time they were dismissed up to their actual reinstatement is proper, conformably
with Art 279 Labor Code.

PHILIPPINE VILLAGE HOTEL V. NLRC 230 SCRA 423


FACTS: Private respondents were employees of petitioner Phil Village Hotel. However
in 1986, petitioner had to close and totally discontinue its operations due to serious
financial and business reverses resulting in the termination of the services of its
employees
1. Thereafter, Phil Village Hotel Employees and Workers Union filed a complaint for
separation pay, unfair labor practice and illegal lockout
2. LA issued an order declaring the losses suffered by the petitioner to be actual
and genuine to merit the valid termination of its employees but directed the
company to prioritize the private respondents should it resume business in the
future. NLRC affirmed the same
3. Subsequently, the company decided to have a one month dry-run operation to
ascertain the feasibility of resuming its business operations. Petitioner hired
casual workers, including private respondents, for 1 month. After evaluating the
individual performance of all the employees and upon the lapse of the one month
contract, petitioner terminate the services of private respondents
4. Private respondents then filed a complaint for illegal dismissal and unfair labor
practice. LA dismissed the same for lack of merit
5. NLRC reversed the decision and ordered the reinstatement of private
respondents without loss of seniority rights and other privileges
ISSUE: WON PRIVATE RESPONDENTS ARE REGULAR EMPLOYEES
CONSIDERING THAT THE LATTERS SERVICES WERE ALREADY PREVIOUSLY
TERMINATED IN 1986 AND THEIR EMPLOYMENT CONTRACTS SPECIFICALLY
PROVIDED FOR ONLY A TEMPORARY ONE-MONTH PERIOD
HELD: The fact that private respondents were required to render services usually
necessary or desirable in the operation of petitioner's business for the duration of the
one (1) month dry-run operation period does not in any way impair the validity of the
contractual nature of private respondents' contracts of employment which specifically
stipulated that the employment of the private respondents was only for one month.
That the decisive determinant in term employment should not be the activities that the
employee is called upon to perform, but the day certain agreed upon by the parties for
the commencement and termination of their employment relationship, a day certain
being understood to be that which must necessarily come, although it may not be
known when. The term period was further defined to be the length of existence;
duration. A point of time marking a termination as of a cause or an activity; an end, a
limit, a bound; conclusion; termination. A series of years, months or days in which
something is completed. A time of definite length or the period from one fixed date to
another fixed date.
In Brent School v. Zamora, the SC enumerated the two guidelines by which fixed
contracts of employments can be said NOT to circumvent security of tenure:

1. The fixed period of employment was knowingly and voluntarily agreed


upon by the parties, without any force, duress or improper pressure being
brought to bear upon the employee and absent any other circumstances
vitiating his consent; or
2. It satisfactorily appears that the employer and employee dealt with each
other on more or less equal terms with no moral dominance whatever
being exercised by the former on the latter."
In the instant case, private respondents were validly terminated by the petitioner when
the latter had to close its business due to financial losses. Following the directives of the
NLRC to give priority in hiring private respondents should it resume its business,
petitioner hired private respondents during their one (1) month dry-run operation.
However, this does not mean that private respondents were deemed to have continued
their regular employment status, which they had enjoyed before their aforementioned
termination due to petitioner's financial losses.
NLRC cannot anymore argue that the temporary cessation of the petitioner's operation
due to financial reverses merely suspended private respondents' employment. The
employee-employer relationship had come to an end when the employer had closed its
business and ceased operations. The hiring of new employees when it re-opened after
3 years is valid and to be expected. The prior employment which was terminated cannot
be joined or tacked to the new employment for purposes of security of tenure.

RAYCOR V. NLRC 261 SCRA 589 (1996)


FACTS: Petitioner is engaged in installing airconditioning systems in the buildings of its
clients. In connection with such installation work, petitioner hired private respondents
who worked in various capacities as tinsmith, leadman, aircon mechanic, installer,
welder and painter.
1. Private respondents insists that they had been regular employees all along, but
petitioner maintains they were project employees who were assigned to work in
specific projects of petitioner, and that the nature of Raycors business prevented
petitioner from hiring private respondents as regular employees
2. Private respondents filed a complaint for regularization but was dismissed for
want of cause of action
3. Private respondents were subsequently dismissed; as a result, private
respondents filed a complaint for illegal dismissal
4. LA dismissed the complaint for lack of merit citing that evidence showed that the
private respondents were project employees within the meaning of Policy
Instructions no 20
5. NLRC reversed the LA decision and found private respondents to have been
regular employees who were illegally dismissed
6. Petitioner anchors its argument on the following:
a. It is engaged solely in the business of installation of air conditioning units or
systems in the buildings of its clients
b. It has no permanent clients with continuous projects where its workers could
be assigned
c. Work is not done in perpetuity but necessarily comes to halt when the
installation of air conditioning units is completed
ISSUE: WERE PRIVATE RESPONDENTS, EMPLOYED BY PETITIONER IN ITS
BUSINESS OF INSTALLING AIRCONDITIONING SYSTEMS IN BUILDINGS,
PROJECT EMPLOYEES OR REGULAR EMPLOYEES
HELD: Private respondents are regular employees.
Private respondents belong to a work pool from which the petitioner drew its manpower
requirements. This is buttressed by the fact that many of the complainants have been
employed for long periods of time already.
Members of a work pool from which the construction company draws its project
employees, if considered employees of the construction company while in the work
pool, are non-project employees or employees for an indefinite period. However, if the
works in the work pool are free to leave any time and offer their services to other
employers then they are project employees employed by a construction company in a
particular project.
Project employees may or may not be members of a work pool, and in turn,
members of a work pool could either be project employees or regular employees.

In the instant case, NLRC did not indicate how private respondents came to be
considered members of a work pool as distinguished from ordinary employees. It did
not establish that a work pool existed in the first place. Neither did it make any finding
as to whether private respondents were indeed free to leave anytime and offer their
services to other employers despite the fact that such determination would have been
critical in defining the precise nature of private respondents employment.
As is evident from the provisions of Art 280 Labor Code, the principal test for
determining whether particular employees are characterized as project employees as
distinguished from regular employees, is whether or not the project employees were
assigned to carry out a specific project or undertaking, the duration (and scope) of
which were specified at the time the employees were engaged for that project.
Considering that in illegal dismissal cases, the employer always has the burden of
proof, and that all doubts be resolved in favor of labor, private respondents are declared
regular employees of petitioner without loss of seniority rights and privileges.

ZAMUDIO V. NLRC MARCH 25, 1990


FACTS: Petitioners rendered services essential for the cultivation of respondents farm.
While the services were not continuous in the sense that they were not rendered
everyday throughout the year, as is the nature of farm work, petitioners had never
stopped working for respondent from year to year from the time he hired them to the
time he dismissed.
ISSUE: ARE PETITONERS CONSIDERED AS EMPLOYEES?
HELD: The nature of their employment, i.e. Pakyao basis, does not make petitioner
independent contractors. Pakyao workers are considered employees as long as the
employer exercises control over the means by which such workers are to perform their
work inside private respondents farm, the latter necessarily exercised control over the
performed by petitioners.
The seasonal nature of petitioners work does not detract from the conclusion that
employer employee relationship exits. Seasonal workers whose work is not merely for
the duration of the season, but who are rehired every working season are considered
regular employees. The circumstances that petitioners do not appears in respondents
payroll does not destroy the employer employee relationship between them. Omission
of petitioners in the payroll was not within their control, they had no hand in the
preparation of the payroll. This circumstance, even if true, cannot be taken against
petitioners.

ECHEVARRRIA V. VENUTEK MEDIKA 516 SCRA 72 (2007)


FACTS: Petitioner Echevarria was an assistant manager of petitioner Venutek Medika,
a corporation engaged in the business of trade and distribution of hospital supplies and
equipment and an affiliate of the Dispophil Group
1. As a matter of policy, the marketing personnel in the Dispophil Group hold a joint
marketing cut-off monthly meeting review the sales and marketing performance
of the companies. Vinuya, as assistant regional sales manager, is in charge of
conducting the monthly meetings
2. During the meeting, Echavarria made disparaging remarks about the senior
offices of the respondent company
3. Respondent was thus issued a memorandum requiring the petitioner to explain in
writing why no disciplinary action should be taken against him. Petitioner denied
the allegations against him
4. Petitioner was again issued a memo requiring him to explain in writing why he
should not be dismissed from employment for violation of Art 282 Labor Code,
specifically: serious misconduct, commission of a crime or offense against the
respondent and willful breach of trust
5. Finding the petitioners explanation unsatisfactory, respondent served upon him a
letter dismissing him from employment effective immediately
6. Petitioner thus filed a complaint for illegal dismissal, nonpayment of salaries and
benefits
7. LA dismissed petitioners complaint for lack of basis and merit
8. NLRC reversed the decision and declared petitioner illegally dismissed

ISSUE: WON PETITIONER IS GUILTY OF SERIOUS MISCONDUCT


HELD: Misconduct has been defined as an improper or wrong conduct; a transgression
of some established and definite rule of action; a forbidden act; a dereliction of duty. It
implies wrongful intent and not mere error of judgment. To be categorized as serious, it
must be of such grave and aggravated character and not merely trivial and unimportant.
And to constitute just cause for an employees separation, it must be in connection with
his work. To justify the termination of an employees services, loss of trust and
confidence as basis thereof must be based on a willful breach of the trust reposed in
him by his employer. Ordinary breach will not suffice.
A breach of trust is willful if it is done intentionally, knowingly and purposely without
justifiable excuse, as distinguished from an act done carelessly, thoughtlessly,
heedlessly or inadvertently.
Petitioners actions no doubt demonstrate a clear disregard for the interests of the
company which he, as a managerial employee, is supposed to safeguard.
That petitioners actions constituted willful breach of trust cannot be gainsaid given his
deliberate choice of an official and company-sanctioned meeting as the venue for

uttering the questioned remarks against Marlene and the use of his position as assistant
marketing manager to gain access to it.
It bears stressing that petitioner was not an ordinary rank-and-file employee. He was a
managerial employee, which required the full trust and confidence of his employer in the
exercise of discretion in overseeing respondents business. As such, he was bound by
more exacting work ethics.

AMA COMPUTER COLLEGE PARANAQUE V. AUSTRIA 538 SCRA 438 (2007)


FACTS: Respondent Austria was hired by AMA Paranaque on probationary
employment as college dean from April 2000 to September 2000. The employment
contract also provides that if Austria gives up the dean position or fails to meet the
standards thereof (based on the evaluation of his immediate superior), he shall be
considered for a faculty position and shall lose the benefits and allowances he enjoys as
dean
1. In August 2000, respondent was charged with violating AMAs employees
handbook of conduct and discipline, particularly: leaking of test questions, failure
to monitor general requirements vital to the operations of the company and gross
inefficiency
2. Thereafter, respondent was placed on preventive suspension and was
subsequently dismissed.
3. Austria, then, filed a complaint for illegal dismissal, illegal suspension,
nonpayment of salary and 13th month pay
4. LA held in favor of respondent Austria, having substantially refuted the charges
against him. However, the LA held that the respondent could no longer be
reinstated beyond September 2000 since his designation as dean was only until
such date.
5. While upholding Austrias dismissal illegal, NLRC held that LA erred in declaring
Austrias employment was only from April to September 2000. NLRC opined that
respondent was a regular employee and that he was illegally dismissed
6. CA affirmed NLRC. It held that based on the handbook and on respondents
appointment, it can be inferred that respondent was a regular employee and as
such his employment can only be terminated for any of the causes under Art 282
Labor Code
7. Petitioner presented the following arguments:
a. Respondent, as college dean, was an academic personnel of AMA under Sec
4(m) (4)(c) of the Manual of Regulations for Private Schools and as such, his
probationary employment is governed by Sec 92 thereof and not by the Labor
Code or AMAs handbook
b. Under the circumstances, respondent has not yet attained the status of a
regular employee
c. Respondent's employment was for a fixed term as found by the Labor Arbiter
but the same was terminated earlier due to just causes;
d. Respondent, whether he may be considered as a probationary or a regular
employee, was dismissed for just causes
ISSUE: WON the respondent is a regular, probationary or fixed term employee
HELD: Respondents employment is one with a fixed term. Art 280 Labor Code does
not proscribe or prohibit an employment contract with a fixed period. As held in Brent
School v. Zamora: That the duties of an employee entail "activities which are usually
necessary or desirable in the usual business or trade of the employer does not
necessarily follow that the employer and employee should be forbidden to stipulate any

period of time for the performance of those activities. There is nothing essentially
contradictory between a definite period of an employment contract and the nature of the
employee's duties set down in that contract as being "usually necessary or desirable in
the usual business or trade of the employer." The concept of the employee's duties as
being "usually necessary or desirable in the usual business or trade of the employer" is
not synonymous with or identical to employment with a fixed term. Logically, the
decisive determinant in term employment should not be the activities that the employee
is called upon to perform, but the day certain agreed upon by the parties for the
commencement and termination of their employment relationship, a day certain being
understood to be "that which must necessarily come, although it may not be known
when."
The instant case involves respondent's position as dean, and comes within the purview
of the Brent School doctrine.
1. The letter of appointment was clear. Respondent was confirmed as Dean of AMA
College, Paraaque, effective from April 17, 2000 to September 17, 2000. In
numerous cases decided by this Court, we had taken notice, that by way of
practice and tradition, the position of dean is normally an employment for a fixed
term.
2. The fact that respondent voluntarily accepted the employment, assumed the
position, and performed the functions of dean is clear indication that he
knowingly and voluntarily consented to the terms and conditions of the
appointment, including the fixed period of his deanship. Other than the
handwritten notes made in the letter of appointment, no evidence was ever
presented to show that respondents consent was vitiated, or that respondent
objected to the said appointment or to any of its conditions. Furthermore, in his
status as dean, there can be no valid inference that he was shackled by any form
of moral dominance exercised by AMA and the rest of the petitioners.
3. the Manual of Regulations for Private Schools Annotated, 38 which provides that
the college dean is the senior officer responsible for the operation of an
academic program, the enforcement of rules and regulations, and the supervision
of faculty and student services. It is well-settled that the position of dean is
primarily academic and, as such, he is considered a managerial employee.
WON respondent was illegally dismissed and is thus entitled to his benefits as
dean
Thus, the unanimous finding of the Labor Arbiter, the NLRC and the CA that respondent
adequately refuted all the charges against him assumes relevance only insofar as
respondents dismissal from the service was effected by petitioners before expiration of
the fixed period of employment. True, petitioners erred in dismissing the respondent,
acting on the mistaken belief that respondent was liable for the charges leveled against
him. But respondent also cannot claim entitlement to any benefit flowing from such
employment after September 17, 2000, because the employment, which is the source of
the benefits, had, by then, already ceased to exist.

ANDERSON V. NLRC 252 SCRA 116 (1996)


FACTS: Petitioner was recruited by Pacific Business Ventures Inc to work as foreman
in the Fiberglass division of Bitar Metal Fabrication factory in Saudi Arabia. The period
of employment was for 2 years starting February 1988
1. 9 months on the job, petitioner was informed by the proprietor and GM that his
services were being terminated. Subsequently, he was deported to the
Philippines
2. Petitioner, then, filed with POEA a complaint for illegal dismissal, recovery of
salary differential, vacation leave pay, refund of transportation expenses and
recruitment violations
3. Private respondents (Pacific Business Ventures and Kamal Al Bitar) alleged that
petitioner had been dismissed for loss of confidence that petitioner lacked the
leadership skills as head of the fiberglass division
4. POEA found Anderson to have been illegally dismissed and ordered private
respondents to pay the balance of petitioners salary for 2 years and salary
differential
5. NLRC set aside the POEA decision and dismissed the petitioners complaint
ISSUE: WON petitioner was illegally dismissed; WON he is entitled to payment of
salaries corresponding to the unexpired portion of his contract
HELD: Petitioner was illegally dismissed.
There is no dispute that loss of confidence constitutes a just cause for terminating an
employer-employee relationship. Proof beyond reasonable doubt is not even required to
terminate employment on this ground. But the loss of confidence cited in this case to
justify the dismissal of petitioner is not based on any act of dishonesty or disloyalty on
the part of petitioner but on alleged lack of leadership, and technical know-how and
on the allegation that worse, he exhibited a negative attitude toward his work.
Kamal Al Bitars cites no specific acts or omissions constituting unsatisfactory
performance by petitioner of his work. What qualities of leadership and technical
knowledge petitioner was required to possess as supervisor of a fiberglass
company has not been specified. There are no specific instances cited to show
petitioners negative attitude toward his work. Indeed, Kamal Al Bitars affidavit
contained nothing but general, vague and amorphous allegations of petitioners
unfitness for the job.
The rule is that an employee cannot be dismissed except for cause as provided by law
and only after due notice and hearing. If an employee is dismissed without cause, he
has a right to be reinstated without loss of seniority rights and other privileges and to be
paid full back wages, inclusive of allowances and other benefits. If he is dismissed
without notice and hearing although for a just cause, he will be entitled to the payment
of indemnity.
If the contract is for a fixed term and the employee is dismissed without just

cause, he is entitled to the payment of his salaries corresponding to the


unexpired portion of his contract. In this case, as petitioners contract was for
two years and his dismissal was not for a just cause, he is entitled to be paid his
salary for 15 months corresponding the balance of the contract. The grant to him
of a termination pay under his employment contract may be considered
indemnity for his dismissal without prior notice and hearing.

Vous aimerez peut-être aussi